100% found this document useful (3 votes)
3K views149 pages

Nislt Professional Exams Past Questions and Answers (Updated Copy)

The document contains past exam questions and answers for the Nigerian Institute of Science Laboratory Technology, compiled for study purposes. It includes various sections covering health personnel, healthcare responsibilities, and English language comprehension. The material is copyrighted and intended for individual use only, with a request for user feedback on its usefulness.

Uploaded by

enidiva10
Copyright
© © All Rights Reserved
We take content rights seriously. If you suspect this is your content, claim it here.
Available Formats
Download as PDF, TXT or read online on Scribd
100% found this document useful (3 votes)
3K views149 pages

Nislt Professional Exams Past Questions and Answers (Updated Copy)

The document contains past exam questions and answers for the Nigerian Institute of Science Laboratory Technology, compiled for study purposes. It includes various sections covering health personnel, healthcare responsibilities, and English language comprehension. The material is copyrighted and intended for individual use only, with a request for user feedback on its usefulness.

Uploaded by

enidiva10
Copyright
© © All Rights Reserved
We take content rights seriously. If you suspect this is your content, claim it here.
Available Formats
Download as PDF, TXT or read online on Scribd

NIGERIAN INSTITUTE OF SCIENCE LABORATORY TECHNOLOGY

EXAMS PAST QUESTIONS AND ANSWERS

RESEARCHED AND COMPILED BY STUDYMATERIALS HR SERVICES

This material is a product of JobsGivers, it’s solely for study purposes, hence buyers are not permitted
to redistribute, sold or alter the content. Offenders are punishable by the copyright law of the federal
republic of Nigeria. For more exams material, recruitment, and other outsourcing services, visit
jobsgivers.com, call our hotline: 08083053951 or click here to chat with us on whatsapp.

JOBSGIVERS.COM

M
PRODUCT REVIEW

O
We have put in so much effect in this material, please if you find it useful, kindly
revisit the product page on our job portal to give us 5 star rating. This will really

C
enable other candidates make decision in buying this material the way you bought

S.
it. You can CLICK HERE TO VISIT THE PRODUCT PAGE, when you get there click on
the ADD REVIEW button. Write a brief review and give your rating.
ER
IV

SESSION A -PREAMBLES
G

1. One of the following health personnel below deals with preventing disease and promoting
BS

health in the society?


JO

A. Public health officer


B. Medical doctor
C. Nurse
D. Lab scientist

2. One of the following healthcare officers deals with health from the perspective of
individuals?

A. Mid-wife
B. Health Councilor
C. Community health office
D. Medical doctor
3. Which is the Nigeria largest and most important government-run, public health agencies?

A. National Agency for Food and Drug Administration and Control (NAFDAC)
B. Teaching hospitals
C. Society for Family health
D. Nigeria Centers for Disease Control (NCDC)

4. What is the average annual GDP of Lagos State?

A. 520 Billion Naira


B. 1 Trillion Naira
C. 3 Trillion Naira
D. 900 Billion Naira

M
5. A patient who is homeless has been admitted to the hospital with multiple health care

O
problems. The nursing case manager, physician, social worker, and clinical dietician work
together to care for the patient. This inter-professional interaction is referred to as?

C
A. Coordination
B. Continuity
C. Collaboration
S.
ER
D. Consultation
IV

6. In the health sector, which type of authorization is generated when services are rendered?
G

A. Retrospective
BS

B. Prospective
C. Concurrent
D. Sub-authorization
JO

7. Managing clinical outcomes, studying variance trends, and evaluating actions taken to correct
deviations from critical pathways are components of?

A. Utilization review
B. Risk management
C. The nursing process
D. Performance improvement

8. A 15-year-old single mother has a three-month-old infant. If the infant needs to be


hospitalized for a non-emergent illness, the consent for admission and treatment must be
signed by who?

A. infant's mother
B. infant's grandparent
C. mother's attorney
D. mother's legal guardian

9. It is estimated that 60 to 70 percent of all decisions regarding a patient's diagnosis, treatment,


hospital admission, and discharge are based on the report of?

A. medical laboratory scientists


B. Nurses
C. Mid-wives

M
D. The patient

O
C
10. One of the options below is the most outstanding quality of a good health worker?

A. Fitness and Agility


B. Empathy and Compassion
S.
ER
C. Multitasking ability
D. Passion for the job
IV

11. The activities of one of the ministries below were formally handled by the Lagos State
G

Ministry of Health at the point of establishment?


BS

A. Lagos state ministry of internal affair


B. Lagos state ministry of education
JO

C. Lagos state ministry of works


D. Lagos state ministry of youth and social development

12. Who is currently the chairman of LAGOS STATE HEALTH MANAGEMENT AGENCY?

A. Dr. Bayo Aderiye


B. Mrs Kemi Ogunyemi
C. Mr Segun Onakoya
D. Dr. Henry Ajiboye

13. Who is currently the director, medical service of LAGOS STATE HEALTH MANAGEMENT AGENCY?
A. Dr. Adeleke Bidemi
B. Pharm. Seun Osikoya
C. Dr. Adebiyi Bolaji
D. Dr. Henry Ajiboye

14. What is the vision statement of the Lagos State Ministry of Health?

A. To provide quality and standard health facilities, and health services that are effective,
efficient and affordable

B. To improve the health status and social economic advancement of individuals and families in
the State using preventive, promotive and curative approaches

C. To attain excellence in health service delivery by applying best practices at all levels of care

M
D. To provide quality health care services that are available, accessible and affordable to its

O
citizens in an equitable manner

C
15. The name Lagos means? S.
ER
A. Lagoon
B. Lakes
C. Rivers
IV

D. Ocean
G
BS

16. The name Lagos was given to the settlement by?

A. An American
JO

B. A Portuguese
C. A British
D. An Indian

17. Select the correct tense in the sentence below?

Segun had been working in a bank for some years

A. Simple past
B. Past continuous
C. Past perfect
D. Past perfect continuous
18. Find the nth term of the given progression 36, 12, 4,.............

A. 14n + 1
B. 12n – 1
C. 12n + 1
D. 14n -1

19. One of the options below is not part of the responsibilities of Lagos State Health Service
Commission?

A. Award medical license to doctors and other healthcare workers in public hospital
B. Implement establishment policies in respect of Public hospital employees
C. Monitor activities of public hospital on appointment, discipline and promotion

M
D. Maintain up to date personnel records of all public hospital employees

O
20. A virus / disease is regarded as pandemic if it_____________?

C
A. Has no known cure
B. Affects all age groups
C.
D.
Ravages the entire world S.
It transfer very fast and has a high mortality rate
ER
IV

ANSWERS TO SESSION A
G
BS

ANSWERS
JO

1. A 2. D 3. D 4. B 5. C

6. C 7. D 8. B 9. A 10. B

11. D 12. A 13. C 14. C 15. B

16. B 17. D 18. B 19. A 20. C


ENGLISH LANGUAGE

1 Choose the option that best conveys the meaning of the underlined
portion in the following sentence;

In the match against the uplanders team, the sub mariners turned out to be the
dark horse

M
• A. played most brilliantly

O
• B. played below their usual form

C
• C. won unexpectedly
• D. lost as expected
• E. won as expected S.
ER
Correct Answer: Option C
IV

2 Choose the option that best conveys the meaning of the underlined portion in
the following sentence;
G

Only the small fry get punished for such social misdemeanors
BS

• A. small boys
• B. unimportant people
JO

• C. frightened people
• D. frivolous people
• E. inexperienced people

Correct Answer: Option B

3 Choose the option that best conveys the meaning of the underlined portion in
the following sentence;

He spoke with his heart in his mouth


• A. courageously
• B. with such unusual cowardice
• C. with a lot of confusion in his speech
• D. without being able to make up his mind
• E. with fright and agitation

Correct Answer: Option E

4 Choose the option that best conveys the meaning of the underlined
portion in the following sentence;

M
The leader in today's issue of our popular newspaper focuses on inflation

O
• A. president
• B. headline

C
• C. editorial


D. columnist
E. proprietor
S.
ER
Correct Answer: Option C
IV

5 Choose the option that best conveys the meaning of the underlined portion
G

in the following sentence;


BS

From the ways my friend talks, you can see he is such a bore

• A. rude
JO

• B. brilliant
• C. uninteresting
• D. overbearing
• E. humorous

Correct Answer: Option C

6 Choose the option that best conveys the meaning of the underlined portion in
the following sentence;
His jail terms were to run concurrently
• A. simultaneously
• B. uniformly
• C. laboriously
• D. consecutively
• E. judiciously

Correct Answer: Option A

7 Choose the option that best conveys the meaning of the underlined
portion in the following sentence;

M
There is some obvious symmetry in the whole presentation

O
• A. confusion
• B. hesitation

C
• C. excitement


D. orderliness
E.dissatisfaction
S.
ER
Correct Answer: Option D
IV
G

8 Choose the option that best conveys the meaning of the underlined portion in
the following sentence;
BS

The bill has to wait as we are now insolvent

• A. overworked
JO

• B. bankrupt
• C. unsettled
• D. insoluble
• E. affluent

Correct Answer: Option B

9 Choose the option that best conveys the meaning of the underlined portion in
the following sentence;

All his plans fell through


• A. failed
• B. were accomplished
• C. had to be reviewed
• D. were rejected
• E. fell

Correct Answer: Option A

10 Choose the option that best conveys the meaning of the underlined portion in
the following sentence;
The balance sheet at the end of the business year shows that we broke even

M
• A. lost heavily

O
• B. made profit

C
• C. neither lost nor gained
• D. had no money to continue business

S.
E. were heavily indebted to our bankers
ER
Correct Answer: Option C
IV

11 Choose the option that best conveys the meaning of the underlined portion in
the following sentence;
G

He was appointed specifically to put the recruits through


BS

• A. assign them to work


• B. train them
JO

• C. discipline them
• D. assist them at work
• E. supervise them at work

Correct Answer: Option B

12 Choose the option that best conveys the meaning of the underlined portion in
the following sentence;

The result of his experiment represents a breakthrough in medical science


• A. an outstanding success
• B. catastrophe
• C. an end to such experiments
• D. breaking point
• E. a colossal failure

Correct Answer: Option A

13 Choose the option that best conveys the meaning of the underlined portion in
the following sentence;

M
Being an optimist, our professor always sees the bright side of most things

O
• A. charming
• B. illumined

C
• C. brilliant


D. pleasing
E. cheerful
S.
ER
Correct Answer: Option E
IV

14 Choose the option that best conveys the meaning of the underlined portion in
G

the ffollowing sentence;


The state government appointed a commission of inquiry to go into the
BS

community's complaints carefully and without prejudice

• A. investigate
JO

• B. search
• C. look for
• D. account for
• E. ascertain

Correct Answer: Option A

15 Choose the option that best conveys the meaning of the underlined portion in
the following sentence;
The traffic situation in Lagos can lead to disastrous consequences; a man lost a
very lucrative job because he was held up by it
• A. confused
• B. delayed
• C. annoyed
• D. intrigued
• E. obstructed

Correct Answer: Option B

16 Choose the option that best conveys the meaning of the underlined portion in
the following sentence;

M
Mrs. Dada has been deserted by her husband because he feels she has a heart of
stone

O
• A. she has a heavy heart

C
• B. she has little warmth of feeling


C. she has a hard heart
D. she is hard hearted
S.
ER
• E. she has a strong heart

Correct Answer: Option D


IV
G

17 Choose the option that best conveys the meaning of the underlined portion in
the following sentence;
BS

Mr. Jack could be a successful businessman if he paid more attention to the


more intricate aspects of his account
JO

• A. Mr Jack will undoubtedly succeed


• B. Mr Jack cannot succeed
• C. Jack will have a very good chance of succeeding
• D. Mr. Jack will find it difficult to succeed
• E. Mr Jack will succeed in spite of all odds

Correct Answer: Option C

18 If my father had not arrived, I would have starved. This sentence means

• A. My father did arrive and I didn't starve


• B. I had to starve because my father didn't come
• C. my father didn't arrive and I didn't starve
• D. I should have starved but I didn't
• E. I had starved before my father arrived

Correct Answer: Option A

19 Choose the option that best conveys the meaning of the underlined portion in
the following sentence;

M
The two sprinters were running neck and neck

O
• A. exact level
• B. very slowly

C
• C. very fast


D. with their necks together
E. together
S.
ER
Correct Answer: Option A
IV

20 Choose the option that best conveys the meaning of the underlined portion in
G

the following sentence;


BS

He stared at her

• A. glanced
JO

• B. peeped
• C. looked
• D. gazed
• E. fixed

Correct Answer: Option D

21 Choose the option that best conveys the meaning of the underlined portion in
the following sentence;
When the man was caught by police he presented a bold front
• A. he attacked the policemen boldly
• B. he walked up to the policemen
• C. he faced the situation with apparent boldness
• D. he bravely attempted to give them a present
• E. he frowned at them in a defiant manner

Correct Answer: Option C

22 Choose the option that best convey the meaning of the underlined portion in
each of the following sentence;
I have been able to observe him at close quarters

M
• A. in government house nearby

O
• B. within a short space of time

C
• C. in a small family house
• D. at close range
• E. at regular intervals S.
ER
Correct Answer: Option D
IV
G

23 Choose the option that best conveys the meaning of the underlined portion in
each of the following sentence;
BS

At the site of the ghastly accident the poor woman's hair stood on end

• A. the woman's hair became straight


JO

• B. the wind blew her hair upwards


• C. the woman was unmoved
• D. the woman disarranged her hair
• E. the woman was frightened

Correct Answer: Option E

24 Complete each of the following sentences by choosing the option that most
suitably fills the space;

When the beggar was tired he ..... down by the roadside


• A. lied
• B. laid
• C. layed
• D. lay
• E. lain

Correct Answer: Option D

25 Complete each of the following sentences by choosing the option that most
suitably fills the space;

M
He did not like .... leaving the class early

O
• A. we
• B. us

C
• C. our


D. ourselves
E. our selves
S.
ER
Correct Answer: Option B
IV

26 Complete each of the following sentences by choosing the option that most
G

suitably fills the space;


BS

Before the operation, the dentist found that his patient's teeth....

• A. have long decayed


JO

• B. have long been decayed


• C. have long being decayed
• D. had long decayed
• E. had been decayed

Correct Answer: Option D

27 Complete each of the following sentences by choosing the option that most
suitably fills the space;
The employer, not the salesman and his representative .... responsible for the
loss
• A. are
• B. are being
• C. are never
• D. have been
• E. is

Correct Answer: Option E

28 Complete each of the following sentences by choosing the option that most
suitably fills the space;
The boy was born before his parents actually got married and so the court has

M
declared him ....

O
• A. illegal

C
• B. illegitimate
• C. illicit


D. unlawful
E. untenable
S.
ER
Correct Answer: Option B
IV

29 As he was still owing me two naira, i was careful not to .... him any more
G

money
BS

• A. lend
• B. loan
• C. borrow
JO

• D. extend
• E. credit

Correct Answer: Option A

30 Complete each of the following sentences by choosing the option that most
suitably fills the space;
Last week the .... at the theater was the longest i had ever seen

• A. assembly
• B. congregation
• C. convention
• D. audience
• E. crowd

Correct Answer: Option D

31 Complete each of the following sentences by choosing the option that most
suitably fills the space;
If you travel by air you will be given an allowance to cover ....

• A. a luggage

M
• B. luggage
• C. luggages

O
• D. a baggage

C
• E. baggages

Correct Answer: Option B


S.
ER
32 Complete each of the following sentences by choosing the option that most
suitably fills the space;
IV

Tennyson and Browning lived about the same time and therefore .... poets
G

• A. contrary
• B. contractual
BS

• C. contemporaneous
• D. contemporary
• E. contemptible
JO

Correct Answer: Option D

33 Complete each of the following sentences by choosing the option that most
suitably fills the space;
The green snake which lives in the green grass is using a perfect ....

• A. ambush
• B. under cover
• C. camouflag
• D. shelter
• E. subterfuge
Correct Answer: Option C

34 Complete each of the following sentences by choosing the option that most
suitably fills the space;
In order to carry out necessary examination the dead body was ....

• A. extracted
• B. extradited
• C. exiled
• D. expelled
• E. exhumed

M
Correct Answer: Option E

O
35 Complete each of the following sentences by choosing the option that most
suitably fills the space;

C
Remember this matter is strictly .....
S.
ER
• A. between you and i
• B. among you and me
• C. among you and me
IV

• D. between you and me


• E. between you and myself
G

Correct Answer: Option D


BS

36 Complete each of the following sentences by choosing the option that most
suitably fills the space;
JO

He went abroad with a view .... a business partner

• A. to find
• B. to be finding
• C. to have found
• D. to finding
• E. he will find

Correct Answer: Option D


37 Complete each of the following sentences by choosing the option that most
suitably fills the space;
He had just entered the office .... the telephone rang

• A. as
• B. before
• C. since
• D. when
• E. until

Correct Answer: Option D

M
38 Complete each of the following sentences by choosing the option that most

O
suitably fills the space;

C
They .... receive visitors on short notice.

• A. use to S.
ER
• B. are used to
• C. used to
• D. are always
IV

• E. are use to
G

Correct Answer: Option A


BS

39 Complete each of the following sentences by choosing the option that most
suitably fills the space;
JO

His suggestion are completely .... the point and cannot be accepted

• A. to
• B. about
• C. beside
• D. on
• E. under

Correct Answer: Option C


40 Complete each of the following sentences by choosing the option that most
suitably fills the space;

For .... he is a secretary we shall not have correct minutes

• A. because
• B. as long as
• C. so long
• D. in as much
• E. although

Correct Answer: Option B

M
41 Complete each of the following sentences by choosing the option that most

O
suitably fills the space;

C
He went up quickly and returned ....



A. fastest
B. fastly
S.
ER
• C. as fastly
• D. as fast
• E. in fastness
IV

Correct Answer: Option D


G
BS

42 Complete each of the following sentences by choosing the option that most
suitably fills the space;
Invariably, he ends his letters .... amicably
JO

• A. you
• B. your's
• C. yours'
• D. yours
• E. your own

Correct Answer: Option D


43 Complete each of the following sentences by choosing the option that most
suitably fills the space;
If i went to the cinema i .... early

• A. shall return
• B. am returning
• C. would return
• D. had return
• E. may return

Correct Answer: Option C

M
44 Complete each of the following sentences by choosing the option that most

O
suitably fills the space; The traditional folktales form an .... to the book

C
• A. index


B. insertion
C. information
S.
ER
• D. overview
• E. appendix
IV

Correct Answer: Option E


G

45 Complete each of the following sentences by choosing the option that most
BS

suitably fills the space;


the frightening explosion in the factory .... whole wing
JO

• A. washed out
• B. wiped out
• C. rooted out
• D. flushed out
• E. cleared out

Correct Answer: Option B

46 Complete each of the following sentences by choosing the option that most
suitably fills the space;
Our plane was scheduled to .... at 10.00 hrs but it was delayed because of the
bad weather
• A. fly out
• B. take off
• C. start out
• D. shoot off
• E. lift off

Correct Answer: Option B

47 Complete each of the following sentences by choosing the option that most
suitably fills the space;
As luck would have it, the weather brightened almost immediately and after

M
six hours flight we .... at Heathrow Airport

O
• A. touched down

C
• B. dropped
• C. came down


D. flew down
E. stopped
S.
ER
Correct Answer: Option A
IV

48 Complete each of the following sentences by choosing the option that most
G

suitably fills the space;


Before we left the airport, each passenger was made to .... certain form
BS

• A. fill in
• B. write in
JO

• C. enter in
• D. go through
• E. read out

Correct Answer: Option A

49 Complete each of the following sentences by choosing the option that most
suitably fills the space;

My father cannot bear the .... of the concoction that emanates from my mothers
soup pot
• A. sweet
• B. savouring
• C. aroma
• D. smell
• E. scent

Correct Answer: Option C

50 Complete each of the following sentences by choosing the option that most

M
suitably fills the space;
I would not have condescended to appease the traffic policeman but i

O
happened .... traffic regulation

C
• A. to have broken


B. to be breaking
C. to break
S.
ER
• D. to break in
• E. to have evaded
IV

Correct Answer: Option A


G
BS
JO
ADVANCED MATHEMATICS

1 If x2−kx+9=0 has equal roots, find the values of k.

 A. 3, 4
 B. ±3
 C. ±5
 D. ±6

Correct Answer: Option D

2 Find the coefficient of x4 in the expansion of (1−2x)6

 A. -320

M
 B. -240
 C. 240

O
 D. 320

C
Correct Answer: Option C

3 S.
Find the 21st term of the Arithmetic Progression (A.P.): -4, -1.5, 1, 3.5,...
ER
 A. 43.5
 B. 46
 C. 48.5
IV

 D. 51
G

Correct Answer: Option B

4 How many ways can 6 students be seated around a circular table?


BS

 A. 36
 B. 48
JO

 C. 120
 D. 720

Correct Answer: Option C

5 A straight line 2x+3y=6, passes through the point (-1,2). Find the equation of the line.

 A. 2x-3y=2
 B. 2x-3y=-2
 C. 2x+3y=-4
 D. 2x+3y=4

Correct Answer: Option D

JobsGivers.com is the most reliable platform for authentic exam past questions. For more, visit: WWW.JOBSGIVERS.COM, or WhatsApp us: 08083053951
6 Given that f(x)=5x2−4x+3 ,find the coordinates of the point where the gradient is 6.

 A. (4,1)
 B. (4,-2)
 C. (1,4)
 D. (1,-2)

Correct Answer: Option C

7 There are 7 boys in a class of 20. Find the number of ways of selecting 3 girls and 2 boys

 A. 1638

M
 B. 2730
 C. 6006

O
 D. 7520

C
Correct Answer: Option C

8 A fair die is tossed twice. What is its smple size?S.


ER
 A. 6
 B. 12
 C. 36
IV

 D. 48
G

Correct Answer: Option C

9
BS

Face 1 2 3 4 5 6
Frequency 12 18 y 30 2y 45
JO

Given the table above as the result of tossing a fair die 150 times, find the mode.

 A. 3
 B. 4
 C. 5
 D. 6

Correct Answer: Option D

10 Given that a = 5i + 4j and b = 3i + 7j, evaluate (3a ‐ 8b).

 A. 9i + 44j
 B. -9i + 44j

JobsGivers.com is the most reliable platform for authentic exam past questions. For more, visit: WWW.JOBSGIVERS.COM, or WhatsApp us: 08083053951
 C. -9i - 44j
 D. 9i - 44j

Correct Answer: Option C

11 Find the magnitude and direction of the vector p=(5i−12j)

 A. (13, 113.38°)
 B. (13, 067.38°)
 C. (13, 025.38°)
 D. (13, 157.38°)

Correct Answer: Option D

M
12 If P = n2+1:n=0,2,3 and Q = n+1:n=2,3,5, find P∩Q.

O
 A. {5, 10}

C
 B. {4, 6}
 C. {1, 3}
 D. { }
S.
ER
Correct Answer: Option D

13 If α and β are the roots of 2x2−5x+6=0, find the equation whose roots are (α+1) and
IV

(β+1)
A. 2x2−9x+15=0
G

B. 2x2−9x+13=0
BS

C. 2x2−9x−13=0
D. 2x2−9x−15=0
JO

Correct Answer: Option B

14 If α and β are the roots of the equation 2x2+5x+n=0, such that αβ=2 , find the value
of n.

 A. -4
 B. -2
 C. 2
 D. 4

Correct Answer: Option D

15 Solve log2(12x−10)=1+log2(4x+3)

JobsGivers.com is the most reliable platform for authentic exam past questions. For more, visit: WWW.JOBSGIVERS.COM, or WhatsApp us: 08083053951
.

 A. 4.75
 B. 4.00
 C. 1.75
 D. 1.00

Correct Answer: Option B

16 The general term of an infinite sequence 9, 4, -1, -6,... is ur=ar+b Find the values of a and
b.

 A. a = 5, b = 14
B. a = -5, b = 14

M

 C. a = 5, b = -14
 D. a = -5, b = -14

O
C
Correct Answer: Option B

S.
17 How many numbers greater than 150 can be formed from the digits 1, 2, 3, 4, 5 without
repetition?
ER
 A. 91
 B. 191
 C. 291
IV

 D. 391
G

Correct Answer: Option C

Find the radius of the circle x2+y2−8x−2y+1=0


BS

18

 A. 9
B. 7
JO


 C. 4
 D. 3

Correct Answer: Option C

19 In how many ways can the letters of the word 'ELECTIVE' be arranged?

 A. 336
 B. 1680
 C. 6720
 D. 20160

Correct Answer: Option C

JobsGivers.com is the most reliable platform for authentic exam past questions. For more, visit: WWW.JOBSGIVERS.COM, or WhatsApp us: 08083053951
20 Find the equation to the circle x2+y2−4x−2y=0 at the point (1, 3).

 A. 2y - x -5 = 0
 B. 2y + x - 5 = 0
 C. 2y + x + 5 = 0
 D. 2y - x + 5 = 0

Correct Answer: Option A

21 Given that y=x(x+1)2, calculate the maximum value of y.

 A. -2
 B. 0

M
 C. 1
 D. 2

O
Correct Answer: Option B

C
22
S.
The midpoint of M(4, -1) and N(x, y) is P(3, -4). Find the coordinates of N.

A. (2, -3)
ER

 B. (2, -7)
 C. (-1, -3)
 D. (-10, -7)
IV

Correct Answer: Option B


G

23 Find the stationary point of the curve y=3x2−2x3


BS

 A. (1, 0)
 B. (-1, 0)
JO

 C. (1, 1)
 D. (-1, -1)

Correct Answer: Option A

24 Calculate the standard deviation of 30, 29, 25, 28, 32 and 24.

 A. 2.0
 B. 2.8
 C. 3.0
 D. 3.2

Correct Answer: Option B

JobsGivers.com is the most reliable platform for authentic exam past questions. For more, visit: WWW.JOBSGIVERS.COM, or WhatsApp us: 08083053951
25 The marks scored by 4 students in Mathematics and Physics are ranked as shown in the
table below

Mathematics 3 4 2 1
Physics 4 3 1 2

Calculate the Spearmann's rank correlation coefficient.

 A. 0.2
 B. 0.5
 C. 0.6
 D. 0.7

M
Correct Answer: Option C

O
26 A body of mass 28g, initially at rest is acted upon by a force, F Newtons. If it attains a
velocity of 5.4ms−1

C
in 18 seconds, find the value of F.

 A. 0.0082N
S.
ER
 B. 0.0084N
 C. 0.082N
 D. 0.084N
IV

Correct Answer: Option B


G

27 Find the angle between forces of magnitude 7N and 4N if their resultant has a magnitude of
9N.
BS

 A. 39.45°
 B. 73.40°
JO

 C. 75.34°
 D. 106.60°

Correct Answer: Option B

28 If P=x:−2<x<5 and Q=x:−5<x<2 are subsets of μ=x:−5≤x≤5, where x is a real


number, find (P∪Q)

A. x:− 5< x < 5


B. x:− 5≤x ≤ 5
C. x:− 5≤ x < 5
D. x:− 5 < x ≤ 5

JobsGivers.com is the most reliable platform for authentic exam past questions. For more, visit: WWW.JOBSGIVERS.COM, or WhatsApp us: 08083053951
Correct Answer: Option A

29 If y=4x−1, list the range of the domain −2≤x≤2, where x is an integer.

 A. {-9, -1, 2,3, 4}


 B. {-9, -2, 0, 1, 7}
 C. {-5, -4, -3, -2}
 D. {-9, -5, -1, 3, 7}

Correct Answer: Option D

30 If the solution set of x2+kx−5=0 is (-1, 5), find the value of k.

M
 A. -6
 B. -4
C. 4

O

 D. 5

C
Correct Answer: Option B

S.
31 Find the equation of the line which passes through (-4, 3) and parallel to line y = 2x + 5.
ER
 A. y = 2x + 11
 B. y = 3x + 11
 B. y = 3x - 5
IV

 C. y = 2x - 11
G

Correct Answer: Option A


BS

32 Points E(-2, -1) and F(3, 2) are the ends of the diameter of a circle. Find the equation of the
circle.

A. x2+y2−5x+3=0
JO

B. x2+y2−2x−6y−13=0
C. x2+y2−x+5y−6=0
D. x2+y2−x−y−8=0

Correct Answer: Option D

33 The lines 2y+3x−16=0 and 7y−2x−6=0 intersect at point P. Find the coordinates of P.

 A. (4, 2)
 B. (4, -2)
 C. (-4, 2)
 D. (-4, -2)

JobsGivers.com is the most reliable platform for authentic exam past questions. For more, visit: WWW.JOBSGIVERS.COM, or WhatsApp us: 08083053951
Correct Answer: Option A

34 Given that n=10 and ∑d2=20, calculate the Spearman's rank correlation coefficient.

 A. 0.121
 B. 0.733
 C. 0.879
 D. 0.979

Correct Answer: Option C

35 A box contains 14 white balls and 6 black balls. Find the probability of first drawing a
black ball and then a white ball without replacement.

M
 A. 0.21
 B. 0.22

O
 C. 0.30
 D. 0.70

C
36
Correct Answer: Option B
S.
Given that r=3i+4j and t=−5i+12j , find the acute angle between them.
ER
 A. 14.3°
 B. 55.9°
IV

 C. 59.5°
 D. 75.6°
G

Correct Answer: Option C


BS

37 A body of mass 10kg moving with a velocity of 5ms−1 collides with another body of mass
15kg moving in the same direction as the first with a velocity of 2ms−1. After collision, the
JO

two bodies move together with a common velocity vms−1

 A. 3.2
 B. 5.3
 C. 7.0
 D. 8.0

Correct Answer: Option A

38 A man of mass 80kg stands in a lift. If the lift moves upwards with acceleration 0.5ms−2,
calculate the reaction from the floor of the lift on the man. [g=10ms−2]

 A. 760N

JobsGivers.com is the most reliable platform for authentic exam past questions. For more, visit: WWW.JOBSGIVERS.COM, or WhatsApp us: 08083053951
 B. 800N
 C. 805N
 D. 840N

Correct Answer: Option D

39 Given that f:x→x2 and g:x→x+3, where x∈R, find fog(2).

 A. 25
 B. 9
 C. 7
 D. 5

Correct Answer: Option A

M
40 In how many ways can a committee of 5 be selected from 8 students if 2 particular students

O
are to be included?

C
 A. 20



B. 28
C. 54
D. 58
S.
ER
Correct Answer: Option A
IV

41 If x=i−3j and y=6i+j, calculate the angle between x and y.


G

 A. 60°
 B. 75°
BS

 C. 81°
 D. 85°

Correct Answer: Option C


JO

42 Given that x2+4x+k=(x+r)2+1, find the value of k and r.

 A. k = 5, r = -1
 B. k = 5, r = 2
 C. k = 2, r = -5
 D. k = -1, r = 5

Correct Answer: Option B

JobsGivers.com is the most reliable platform for authentic exam past questions. For more, visit: WWW.JOBSGIVERS.COM, or WhatsApp us: 08083053951
43

Marks 0 1 2 3 4 5
Number of candidates 6 4 8 10 9 3

The table above shows the distribution of marks scored by students in a test. How many
candidates scored above the median score?

 A. 3
 B. 12
 C. 22
 D. 30

M
Correct Answer: Option B

O
44

C
Marks 0123 45
Number of candidates 6 4 8 10 9 3
S.
The table above shows the distribution of marks scored by students in a test. Find the
ER
interquartile range of the distribution.

 A. 4
IV

 B. 3
 C. 2
D. 1
G


BS

Correct Answer: Option B

45 A mass of 75kg is placed on a lift. Find the force exerted by the floor of the lift on the mass
when the lift is moving up with constant velocity. [g=9.8ms−2]
JO

 A. 750N
 B. 745N
 C. 735N
 D. 98N

Correct Answer: Option C

46 Given that y=4−9x and Δx=0.1, calculate Δy

 A. 9.0
 B. 0.9
 C. -0.3

JobsGivers.com is the most reliable platform for authentic exam past questions. For more, visit: WWW.JOBSGIVERS.COM, or WhatsApp us: 08083053951
 D. -0.9

Correct Answer: Option D

47 In calculating the mean of 8 numbers, a boy mistakenly used 17 instead of 25 as one of the
numbers. If he obtained 20 as the mean, find the correct mean.

 A. 24
 B. 23
 C. 21
 D. 19

Correct Answer: Option C

M
48 If 2sin2θ=1+cosθ,0° ≤ θ ≤ 90°, find the value of θ

O
 A. 90°
 B. 60°

C
 C. 45°
 D. 30°

Correct Answer: Option B


S.
ER
49 A 24N force acts on a body such that it changes its velocity from 5m/s to 9m/s in 2 secs.If
the body is travelling in a straight line, calculate the distance covered in the period.
IV

 A. 22m
 B. 18m
G

 C. 14m
 D. 10m
BS

Correct Answer: Option C


JO

50 The sum, Sn, of a sequence is given by Sn=2n2−5. Find the 6th term.

 A. 112
 B. 67
 C. 45
 D. 22

Correct Answer: Option D

51 Forces F1=(8N,030°) and F2=(10N,150°) act on a particle. Find the horizontal


component of the resultant force.

 A. 1.7N

JobsGivers.com is the most reliable platform for authentic exam past questions. For more, visit: WWW.JOBSGIVERS.COM, or WhatsApp us: 08083053951
 B. 4.5N
 C. 9.0N
 D. 13.0N

Correct Answer: Option A

52 If f(x)=3x3+8x2+6x+k and f(2)=1, find the value of k.

 A. -67
 B. -61
 C. 61
 D. 67

Correct Answer: Option A

M
53 If (x+2) and (3x−1) are factors of 6x3+x2−19x+6 , find the third factor.

O
A. 2x−3

C
B. 3x+1
C. x−2
D. 3x+2 S.
ER
Correct Answer: Option A
IV

54 If 2,(k+1),8,...form an exponential sequence (GP), find the values of k.


G

 A. -3 and 5
BS

 B. 5 and -5
 C. 3 and -3
 D. -5 and 3
JO

Correct Answer: Option D

55 A box contains 5 red and k blue balls. A ball is selected at random from the box. If the
probability of selecting a blue ball is , find the value of k.

 A. 5
 B. 6
 C. 8
 D. 10

Correct Answer: Option D

JobsGivers.com is the most reliable platform for authentic exam past questions. For more, visit: WWW.JOBSGIVERS.COM, or WhatsApp us: 08083053951
56

Age in years 10 - 14 15 - 19 20 - 24 25 - 29 30 - 34
Frequency 6 8 14 10 12

What is the class mark of the median class?

 A. 17
 B. 22
 C. 27
 D. 32

Correct Answer: Option B

M
57

O
Age in years 10 - 14 15 - 19 20 - 24 25 - 29 30 - 34

C
Frequency 6 8 14 10 12

In which group is the upper quartile? S.


ER
 A. 15 - 19
 B. 20 - 24
 C. 25 - 29
IV

 D. 30 - 34
G

Correct Answer: Option C

58
BS

Age in years 10 - 14 15 - 19 20 - 24 25 - 29 30 - 34
Frequency 6 8 14 10 12
JO

Find the mean of the distribution.

 A. 23.4
 B. 23.6
 C. 24.3
 D. 24.6

Correct Answer: Option A

59 Find the angle between (5i+3j) and (3i−5j)

JobsGivers.com is the most reliable platform for authentic exam past questions. For more, visit: WWW.JOBSGIVERS.COM, or WhatsApp us: 08083053951
 A. 180°
 B. 90°
 C. 45°
 D. 0°
 D. 0°

Correct Answer: Option B

60 Find the coefficient of x3 in the binomial expansion of (3x+4)4 in ascending powers of x.

 A. 432
 B. 194
 C. 144
 D. 108

M
Correct Answer: Option A

O
61 The deviations from the mean of a set of numbers are (k+3)2,(k+7),−2,k and

C
(k+2)2, where k is a constant. Find the value of k.
 A. 3
S.
ER
 B. 2
 C. -2
 D. -3
IV

Correct Answer: Option D


G

62 The first term of a linear sequence is 9 and the common difference is 7. If the nth term is
380, find the value of n.
BS

 A. 45
 B. 54
JO

 C. 56
 D. 65

Correct Answer: Option B

63 For what values of m is 9y2 + my + 4 a perfect square?


A. ±2
B. ±3
C. ±6
D. +12

Correct Answer: Option D

JobsGivers.com is the most reliable platform for authentic exam past questions. For more, visit: WWW.JOBSGIVERS.COM, or WhatsApp us: 08083053951
64 In how many ways can 9 people be seated on a bench if only 3 places are available?

 A. 1200
 B. 504
 C. 320
 D. 204

Correct Answer: Option B

65 If the points (-1, t -1), (t, t - 3) and (t - 6, 3) lie on the same straight line, find the values of t.

 A. t = -2 and 3
 B. t = 2 and -3
 C. t = 2 and 3

M
 D. t = -2 and -3

O
Correct Answer: Option C

C
66 A ball is thrown vertically upwards with a velocity of 15ms−1. Calculate the maximum
height reached. [g=10ms−2]

A. 15.25m
S.
ER

 B. 13.25m
 C. 11.25m
 D. 10.25m
IV

Correct Answer: Option C


G

67 Find the distance between the points (2, 5) and (5, 9).
BS

 A. 4 units
 B. 5 units
 C. 12 units
JO

 D. 14 units

Correct Answer: Option B

68 Find the equation of the line that is perpendicular to 2y+5x−6=0 and bisects the line
joining the points P(4, 3) and Q(-6, 1).

 A. y + 5x + 3 = 0
 B. 2y - 5x - 9 = 0
 C. 5y + 2x - 8 = 0
 D. 5y - 2x - 12 = 0

Correct Answer: Option D

JobsGivers.com is the most reliable platform for authentic exam past questions. For more, visit: WWW.JOBSGIVERS.COM, or WhatsApp us: 08083053951
69 An object is thrown vertically upwards from the top of a cliff with a velocity of
25ms−1. Find the time, in seconds, when it is 20 metres above the cliff. [g=10ms−2]

 A. 0 and 1
 B. 0 and 4
 C. 0 and 5
 D. 1 and 4

Correct Answer: Option D

70 The angle subtended by an arc of a circle at the centre is π3radians. If the radius of the circle
is 12cm, calculate the perimeter of the major arc.

M
A. 4 (6+5π)
B. 4 (6+2π)

O
C. 4 (3+3π)

C
D. 4 (3+5π)

Correct Answer: Option A S.


ER
71 A committee consists of 5 boys namely: Kofi, John, Ojo, Ozo and James and 3 girls
namely: Rose, Ugo and Ama. In how many ways can a sub-committee consisting of 3 boys
and 2 girls be chosen, if Ozo must be on the sub-committee?
IV

 A. 35
G

 B. 30
 C. 18
BS

 D. 12

Correct Answer: Option C


JO

72 The sales of five salesgirls on a certain day are as follows; GH¢ 26.00, GH¢ 39.00, GH¢
33.00, GH¢ 25.00 and GH¢ 37.00. Calculate the standard deviation if the mean sale is GH¢
32.00.

 A. GH¢ 5.65
 B. GH¢ 5.66
 C. GH¢ 6.5
 D. GH¢ 6.56

Correct Answer: Option B

73 Two bodies of masses 3kg and 5kg moving with velocities 2 m/s and V m/s respectively in
opposite directions collide. If they move together after collision with velocity 3.5 m/s in the
direction of the 5kg mass, find the value of V.

JobsGivers.com is the most reliable platform for authentic exam past questions. For more, visit: WWW.JOBSGIVERS.COM, or WhatsApp us: 08083053951
 A. 7.8 m/s
 B. 6.8 m/s
 C. 5.6 m/s
 D. 4.6 m/s

Correct Answer: Option B

74 If s=3i−j and t=2i+3j, find (t−3s).(t+3s)

 A. -77
 B. -71
 C. -53
 D. -41

M
Correct Answer: Option A

O
75 If 2sin2 θ=1+cosθ,0°≤θ≤90°, find θ

C
 A. 30°


B. 45°
C. 60°
D. 90°
S.
ER

Correct Answer: Option C


IV

76 Find the upper quartile of the following scores: 41, 29, 17, 2, 12, 33, 45, 18, 43 and 5.
G

 A. 45
 B. 41
BS

 C. 33
 D. 21

Correct Answer: Option B


JO

77 Calculate the mean deviation of 1, 2, 3, 4, 5, 5, 6, 7, 8, 9.

 A. 2
 B. 3
 C. 4
 D. 5

Correct Answer: Option A

87 Find the equation of the straight line that passes through (2, -3) and perpendicular to the
line 3x - 2y + 4 = 0.

 A. 2y - 3x = 0

JobsGivers.com is the most reliable platform for authentic exam past questions. For more, visit: WWW.JOBSGIVERS.COM, or WhatsApp us: 08083053951
 B. 3y - 2x + 5 = 0
 C. 3y + 2x + 5 = 0
 D. 2y - 3x - 5 = 0

Correct Answer: Option C

88 A body is kept at rest by three forces F1=(10N,030°),F2=(10N,150°) and F3.


Find F3

 A. (12N, 090°)
 B. (10N, 270°)
 C. (10N, 180°)
 D. (10N, 120°)

M
Correct Answer: Option B

O
89 Which of the following sets is equivalent to (P∪Q)∩(P∪Q′)?

C
A. P
B. P∩Q
C. P∪Q
S.
ER
D. ∅

Correct Answer: Option A


IV

90 Solve the inequality x2−2x≥3


G

A. −1≤x≤3
BS

B. x≥3 and x≤−1


C. x≥3 or x<−1
JO

D. −1≤x<3

Correct Answer: Option B

91 If (x + 1) is a factor of the polynomial x3+px2+x+6 . Find the value of p.

 A. -8
 B. -4
 C. 4
 D. 8

Correct Answer: Option B

JobsGivers.com is the most reliable platform for authentic exam past questions. For more, visit: WWW.JOBSGIVERS.COM, or WhatsApp us: 08083053951
92 A polynomial is defined by f(x+1)=x3+px2−4x+2, find f(2).

 A. -8
 B. -2
 C. 2
 D. 8

Correct Answer: Option C

93 The equation of a circle is 3x2+3y2+24x−12y=15. Find its radius.

 A. 2

M
 B. 3
 C. 4
D. 5

O

C
94 If the midpoint of the line joining (1 - k, -4) and (2, k + 1) is (-k, k), find the value of k.



A. -4
B. -3
C. -2
S.
ER

 D. -1

Correct Answer: Option D


IV

95 The marks obtained by 10 students in a test are as follows: 3, 7, 6, 2, 8, 5, 9, 1, 4 and 10.


G

Find the mean mark.


BS

 A. 4.50
 B. 5.50
 C. 6.50
 D. 6.75
JO

Correct Answer: Option B

96 The marks obtained by 10 students in a test are as follows: 3, 7, 6, 2, 8, 5, 9, 1, 4 and 10.


Find the variance.

 A. 8.25
 B. 8.50
 C. 9.00
 D. 9.17

Correct Answer: Option A

JobsGivers.com is the most reliable platform for authentic exam past questions. For more, visit: WWW.JOBSGIVERS.COM, or WhatsApp us: 08083053951
97 If r denotes the correlation coefficient between two variables, which of the following is
always true?

A. 0<r≤1
B. −1 ≤ r < 1
C. −1 < r ≤ 0
D. −1 ≤ r ≤ 1

Correct Answer: Option D

98 A stone is dropped from a height of 45m. Find the time it takes to hit the ground.
[g=10ms−2]

M
 A. 3.0 seconds

O
 B. 4.5 seconds
 C. 5.0 seconds

C
 D. 9.0 seconds

Correct Answer: Option A


S.
ER
99 In computing the mean of 8 numbers, a boy mistakenly used 17 instead of 25 as one of the
numbers and obtained 20 as the mean. Find the correct mean

 A. 19
IV

 B. 21
 C. 23
G

 D. 24
BS

Correct Answer: Option B

100 Which of the following is nor a measure of central tendency?


JO

 A. Mean
 B. Variance
 C. Median
 D. Mode

Correct Answer: Option B

JobsGivers.com is the most reliable platform for authentic exam past questions. For more, visit: WWW.JOBSGIVERS.COM, or WhatsApp us: 08083053951
FOR ALL MEDICAL AND HEALTH OFFICERS POSITIONS

1. DPT vaccination is for immunization against

A. Diphtheria, Pneumonia and Tetanus


B. Diphtheria, Pertussis and Tetanus
C. Diarrhoea, Pneumonia and Tetanus
D. Diphtheria, Pneumonia and Typhoid

M
E. None of these

O
Correct Ans D: Diphtheria, Pneumonia and Typhoid

C
S.
2. Given below are two statements, one labelled as Assertion (A) and the other labelled
as Reason (R):
ER
Assertion (A) : Vitamin B complex is taken along with antibiotics to protect the intestinal
flora of E. coli bacteria.
IV

Reason (R) : Antibiotics kill bacteria in the body


G
BS

In the context of the above two statements, which one of the following is correct?
JO

A. Both A and R are true and R is the correct explanation of A


B. Both A and R are true, but R is not a correct explanation of A
C. A is true, but R is false
D. A is false, but R is true
E. None of these

Correct Ans B : Both A and R are true, but R is not a correct explanation of A

3. People drinking water from a shallow handpump are likely to suffer from all of the
following diseases except

A. Cholera
B. Typhoid
C. Jaundice
D. Fluorosis
E. None of these

Correct Ans C: Jaundice

4. Of the four landmarks in medical history given below, which one was the first to take
place?

A. Organ transplant

M
B. Bypass surgery
C. Test tube baby

O
D. Plastic surgery

C
E. None of these

Correct Ans B:Plastic surgery S.


ER
5. Consider the following statements:
IV

AlDS is transmitted

1. by sexual intercourse- 2. by blood transfusion


G

3. by mosquitoes and other


BS

bloodosuclting insects 4. across the placenta


JO

l,2 and 3 are correct

A. 1,2 and 4 are correct


B. 1,3 and 4 are correct
C. 1 and 3 are correct
D. None of these

Correct Ans A: 1,2 and 4 are correct


6. Rauwolfia serpentina is useful in the control of

A. Tuberculosis
B. Whooping cough
C. Hydrophobia
D. Blood pressure
E. None of these

Correct Ans D:Blood pressure

7. An important herb, found in the forests of Northern Kashmir, Dehradun, Himachal

M
Pradesh and now in cultivation in many other States of india, widely used in cough, sore
throat and bronchitis is

O
A. Emblica

C
B. Liquorice
C.
D.
Worm seed
Chhatim
S.
ER
E. None of these
IV

Correct Ans A: Emblica


G

8. The deficiency of vitamin B in human beings causes


BS

A. Leucopenia
B. Diphtheria
JO

C. Beri-Beri
D. Poliomyelitis
E. None of these
Correct Ans C: Beri-Beri

9. In certain States of india, especially Madhya Pradesh and Uttar Pradesh,' the use of
khesari pulse for a long time led to arthritis and paralysis of the legs, because it

A. is devoid of protein
B. lacks calcium
C. contains (N.Y. L. glittomyle) amino-proprio-nitric
D. contains higher quantity of iron
E. None of these

Correct Ans C: contains (N.Y. L. glittomyle) amino-proprio-nitric

10. “Haemophilia” is a disease in which

A. the eosinophil number drastically increases


B. the haemoglobin content rises
C. the blood loses the property to clot outside the body after an injury
D. respiratory disorder takes place

M
E. None of these

O
Correct Ans C: the blood loses the property to clot outside the body after an
injury

C
S.
11. Which one of the following is the modern tool for imaging brain tumors?
ER
A. X-rays
B. Magnetic resonance
IV

C. Electron spin resonance


D. Ultrasound
G

E. None of these
BS

Correct Ans A:Magnetic resonance


JO

12. Insulin regulates

A. reproduction
B. heart beat
C. blood glucose level
D. physical growth
E. None of these
Correct Ans C:blood glucose level
13. Match List! with List II and select the correct answer by using the codes given below
the lists:

List 1 List ll

A) BCG vaccine 1) Malaria

B) BPL vaccine 2) Sore throat

C) Chloroquine 3) Tuberculosis

M
D) Penicillin 4) Rabies

O
C
A. 3412
B. 3421
C.
D.
4321
4312
S.
ER
E. None of these

Correct Ans B:3 4 1 2


IV
G

14. Who founded the Western Scientific Therapy?


BS

A. Hansen
B. Hippocrates
JO

C. Plato
D. Herodotus
E. None of these

Correct AnsB: Hippocrates

15. If most of the children of a particular village suffered from night blindness, the
nutritional supplement they need to be provided with would include

A. pulses
B. fruits like guava
C. carrots
D. radish
E. None of these
Correct Ans C:carrots

16. If the thyroid of a newborn child is removed, it will cause

A. Gigantism
B. Cretinism
C. only a

M
D. All of these

O
E. None of these
Correct Ans B:Cretinism

C
S.
17. Which of the following disease is caused due to shortage of red blood cells?
ER
A. Meningitis
B. Leukaemia
IV

C. Arthritis
D. Anaemia
G

E. None of these
BS

Correct Ans D:Anaemia


JO

18. Which of the following is a correct description of the use of 'Tetracycline'?

A. It enhances qualities of aeroplane fuel


B. It is known as anti-plague drug
C. It is a preservative
D. It is a pest killer
E. None of these
Correct Ans B: It is known as anti-plague drug
19. What care is most useful to save the life of a child suffering from acute diarrhoea?

A. keep away the child from the gatherings of people


B. do not allow mosquitoes to bite
C. do not give water
D. do not allow the body to dehydrate
E. None of these
Correct Ans D: do not allow the body to dehydrate

20. Which fruit diabetic patients can eat freely?

M
A. Mango
B. Banana

O
C. Orange

C
D. Guava
E. None of these
Correct Ans C: Orange
S.
ER

21. Which one of the following is caused by the expression of recessive gene present on
IV

sex chromosome?
G

A. Rheumatism
BS

B. Nervous shock
C. Muscular dystrophy
D. Cerebral haemorrhage
JO

E. None of these
Correct Ans C: Muscular dystrophy

22. Who founded the Ayurveda system of medicine?

A. Susruta
B. Thirumoolar
C. Atreya
D. Agasthiya
E. None of these
Correct Ans C:Atreya

23. What is endemic disease

A. Disease prevalent in a particular area


B. Disease that attacks many people in one region
C. There is no haemoglobin in the blood
D. None of these
E. All a,b,c
Correct AnsB: Disease prevalent in a particular area

M
O
24. When is the vaccination given to a child?

C
A. Eight months
B.
C.
Within six months of its birth
A year after
S.
ER
D. None of these
E. All a,b,c
IV

Correct Ans B:Within six months of its birth


G
BS

25. What is deficiency disease?

A. Absence of Vitamin
JO

B. Diabetes
C. Phobia
D. None of these
E. All a,b,c
Correct Ans A: Absence of Vitamin

26. What is plastic surgery

A. To set right physical deformity by grafting


B. Surgery done with a plastic instrument
C. To remove decayed parts of the body
D. None of these
E. All a,b,c
Correct Ans A: To set right physical deformity by grafting

27. What is antiseptic?

A. it is a drug which destroys germs


B. It is used for growing rice
C. It is a tablet
D. None of these
E. All a,b,c

M
Correct Ans A: it is a drug which destroys germs

O
C
28. Who was the first female doctor in the world?

A.
B.
Joan of Arc
Elizabeth Blackwell
S.
ER
C. Muthulakshmi Reddy
D. Alva
IV

E. None of these
Correct Ans B: Elizabeth Blackwell
G
BS

29. What is the use of Lithotripter instrument?


JO

A. To remove stones without operation


B. To measure humidity in air
C. To find out whether petrol is adulterated
D. None of these
E. All a,b,c
Correct Ans A:To remove stones without operation

30. Haemophilia is a genetic disorder which causes

A. Blindness
B. Loss of Haemoglobin
C. Rheumatism
D. Non-clotting of blood
E. None of these
Correct Ans D: Non-clotting of blood

31. Which country alone follows Sawdust cure?

A. Japan
B. China
C. Belgium
D. Italy

M
E. None of these
Correct Ans A: Japan

O
C
32. Name the first woman doctor in Tamil Nadu.

A. Dr. Muthulakshmi Reddy


S.
ER
B. Mrs. Tara Cherian
C. Dr. Kanthamani
IV

D. Dr. Annie Besant


E. None of these
G

Correct Ans: A Dr. Muthulakshmi Reddy


BS

33. Lock Jaw i.e. difficulty in opening the mouth is a symptom of


JO

A. Tetanus
B. Cholera
C. Mumps
D. Cancer
E. None of these

Correct Ans A: Tetanus

34. 'Thrombosis' is a disease of the


A. Ear
B. Skin
C. Pituitary
D. Blood
E. None of these
Correct Ans D: Blood

35. What for Nivalin is used?

A. Diabetes
B. Eye disease

M
C. Cancer

O
D. Nerve disease
E. None of these

C
Correct Ans D: Nerve disease
S.
ER
36. Who discovered Kala-azar Fever?

A. Plimsoll
IV

B. Binet
G

C. UN. Brahmachari
D. Banting
BS

E. None of these
Correct Ans C: UN. Brahmachari
JO

37. Which of the following is not a contagious disease?

A. Typhoid
B. Measles
C. Influenza
D. Hysteria
E. None of these
Correct Ans D: Hysteria
38. Laparotomy is associated with

A. female sterilisation
B. male sterilisation
C. brain surgery
D. incision and abdomen
E. None of these
Correct Ans D: incision and abdomen

M
O
39. The instrument used to inspect the lining of the stomach is

C
A. Gastroscope
B.
C.
ECG
Cystoscope
S.
ER
D. Endoscope
E. None of these
Correct Ans A: Gastroscope
IV
G

40. Which organ grows first in a womb?


BS

A. Brain
B. Heart
JO

C. Eye
D. Nose
E. None of these
Correct Ans B: Heart

41. Dialysis is used for the treatment of

A. Kidney failure
B. Heart weakness
C. Brain diseases
D. None of these
E. All a,b,c
Correct Ans A: Kidney failure

42. Who discovered that with the increase of heart beat one's intelligence increases?

A. Dr. Khorana
B. Dr. Rangachary
C. John I. Kokoba
D. Dr. Alexander
E. None of these
Correct Ans C: John I. Kokoba

M
O
44. What is the central telephone exchange of a human body?

C
A. Cerebrum
B.
C.
Cerebellum
Spinal Cord
S.
ER
D. Medulla Oblongata
E. None of these
IV

Correct Ans B: Cerebellum


G

45. Who first dissected the human body to study about its system?
BS

A. Khorana
B. Herophilus
JO

C. Susrutha
D. None of these
E. All a,b,c
Correct Ans B: Herophilus

46. Where was the first medical school opened?

A. Paris
B. Luxembourg
C. Bristol
D. London
E. None of these
Correct Ans C: Bristol

47. Blood does not coagulate inside the body due to the presence of

A. Haemoglobin
B. Fibrin
C. Heparin
D. Plasma
E. None of these
Correct Ans C:Heparin

M
O
48. Nervous disorder is caused by the deficiency of

C
A. B_1
B. B_2
C. B_6 S.
ER
D. B_12
E. None of these
Correct Ans C: B_6
IV
G

49. Who discovered body's reflex action


BS

A. John Hadley
B. Harwood
JO

C. Marshall Hall
D. None of these
E. All a,b,c
Correct Ans C: Marshall Hall

50. What is Cystitis?

A. Inflammation of tongue
B. Inflammation of the urinary bladder
C. Another name for Parkinson's disease
D. None of these
E. All a,b,c
Correct Ans B: Inflammation of the urinary bladder

51. What is myelitis?

A. Inflammation of the spinal cord


B. It is the botanical name for tamarind
C. It is a disease affecting animals in general
D. All of these
E. None of these
Correct Ans A: Inflammation of the spinal cord

M
O
52. Who performed the first open heart surgery?

C
A.
B.
Jenner
Marshall Hall
S.
ER
C. Walton Lillehei
D. None of these
E. All a,b,c
IV

Correct Ans C: Walton Lillehei


G
BS

53. Who was the founder of Modern Anatomy?

A. James Lind
JO

B. Andreas Vesalius
C. Marshall Hall
D. None of these
E. All a,b,c
Correct Ans B: Andreas Vesalius

54. The first homotransplantation of kidney is performed by

A. Barnard
B. Rolf
C. Miller
D. R.H. Lawler
E. None of these
Correct Ans D: R.H. Lawler

55. Who first made a study of digestive system?

A. Geevan Pavlov
B. Sir B.C. Roy
C. Dr. Philip Drucker
D. Eijkman
E. None of these

M
Correct Ans A: Geevan Pavlov

O
C
56. Lack of which vitamin is the cause for mouth ulcer?

A. Vitamin C
S.
ER
B. Vitamin B
C. Vitamin A
D. Vitamin K
IV

E. None of these
G

Correct Ans A: Vitamin C


BS

57. Who prepared Polio Vaccine?


JO

A. Chalin
B. Paul Mueller
C. Jonas E. Salk
D. Louis Pasteur
E. None of these
Correct Ans C: Jonas E. Salk

58. Plague is spread by infected

A. rats
B. pigs
C. mosquitoes
D. houseflies
E. None of these
Correct Ans A: rats

59. Malaria can be cured by administering

A. Quinine
B. Streptomycin
C. Penicillin
D. None of these

M
E. All a,b,c

O
Correct Ans A: Quinine

C
S.
60. Which isotope is used to find out obstruction in the blood stream?
ER
A. Sodium-24
B. Sodium-35
C. Cobalt-22
IV

D. None of these
G

E. All a,b,c
Correct Ans A: Sodium-24
BS

61. Who discovered Beriberi?


JO

A. Macmillan
B. Alexander
C. Eijkman
D. Mostley
E. None of these
Correct Ans C:Eijkman

62. Which one of the following diseases is waterborne?

A. Cholera
B. Kala-azar
C. Filariasis
D. Scurvy
E. None of these
Correct Ans A:Cholera

63. Who was the 'Father of Homeopathy'?

A. Samuel Hahnemann
B. Cockerell
C. F.Banting

M
D. Priestly

O
E. None of these
Correct Ans A: Samuel Hahnemann

C
S.
64. Who first explained the theory of coagulation?
ER
A. Morawitz
B. Alexander Fleming
IV

C. Dr. Khorana
G

D. None of these
E. All a,b,c
BS

Correct Ans A: Morawitz


JO

65. Which disease can be cured by Bacteriophages?

A. Typhoid
B. Arthritis
C. Stomach-ache
D. Cancer
E. None of these
Correct Ans A:Typhoid

66. Which vitamin is present in butter?


A. A
B. B
C. C
D. None of these
E. All a,b,c
Correct Ans A: A

67. Name the bacteria that produces syphilis.

A. Treponema Pallidum
B. Salmonella typhi

M
C. Neisseria Gonorrhoeae

O
D. None of these
E. All a,b,c

C
Correct Ans A: Treponema Pallidum
S.
ER
68. Which vitamin is present in Cauliflower?

A. E
IV

B. K
C. B_1
G

D. B_12
BS

E. None of these
Correct Ans B: K
JO

70. Which mineral is present in banana?

A. Calcium
B. Iron
C. Sodium
D. Potassium
E. None of these
Correct Ans D:Potassium
71. Which country founded Unani system of treatment?

A. Greece
B. Egypt
C. Arabia
D. India
E. None of these
Correct Ans A: Greece

72. How many bones are there in a newly born infant?

M
A. 300
B. 230

O
C. 206

C
D. 280
E. None of these
Correct Ans A: 300 S.
ER

73. Who diagnosed Cancer?


IV

A. Robert Weinberg
G

B. Robert Koch
C. Henry Swan
BS

D. None of these
E. All a,b,c
JO

Correct Ans A: Robert Weinberg

74. Blood circulation was discovered by

A. William Harvey
B. Fleming
C. Louis
D. Hovercraft
E. None of these
Correct Ans A:William Harvey
75. Who has introduced phototherapy?

A. N.R. Flusen
B. Edward Calvin
C. Einthoven
D. Robert
E. None of these
Correct Ans A: N.R. Flusen

76. Who discovered contraceptive pills?

M
A. Dr. Khorana
B. Gregory Pincus

O
C. Kolf

C
D. None of these
E. All a,b,c
Correct Ans B: Gregory Pincus
S.
ER

77. Which part of the body is affected by trachoma?


IV

A. Lungs
G

B. Skin
C. Tongue
BS

D. Eyes
E. None of these
JO

Correct Ans D:Eyes

78. Which part of the body is affected by nephritis?

A. Kidney
B. Skin
C. Spinal cord
D. Brain
E. None of these
Correct Ans A: Kidney
79. Who invented kidney machine?

A. Kolf
B. Koch
C. Hoffman
D. None of these
E. All a,b,c
Correct Ans A: Kolf

80. What is the normal composition of R.B.C in men?

M
A. 4.5 to 6.5 million/aura).
B. 3.9 to 5.5 million/c.m.m

O
C. 2.1 to 2.7 million/c.m.m.

C
D. None of these
E. All a,b,c
S.
Correct Ans A: 4.5 to 6.5 million/aura).
ER

81. Who is Hoffman?


IV

A. Discovered L.S.D.
G

B. Father of Endocrinology
C. Discovered malaria germs
BS

D. None of these
E. All a,b,c
JO

Correct Ans C:Discovered L.S.D.

82. Which organ in the body produces the largest quantity of enzymes?

A. Stomach
B. Liver
C. Thyroid glands
D. None of these
E. All a,b,c
Correct Ans B: Liver
83. Who is the father of neurology?

A. F.Sertumer
B. Franz Joseph Gall
C. Paul Ehrlich
D. None of these
E. All a,b,c
Correct Ans B: Franz Joseph Gall

84. Which is the alternative medicine for Penicillin?

M
A. Erythromycin
B. Aureomycin

O
C. Beldone

C
D. Any Sulpha drugs
E. None of these
Correct Ans A: Erythromycin S.
ER

85. Shortness of breath in heart disease is


IV

A. Dizziness
G

B. Bronchitis
C. Dyspnoea
BS

D. none of these
E. All a,b,c
JO

Correct Ans C:Dyspnoea

86. CHD is a

A. a type of cancer
B. heart disease
C. a disease in liver
D. none of these
E. All a,b,c
Correct Ans B: heart disease
87. The main cause for stroke is

A. Obesity
B. Increased blood pressure
C. Diabetes
D. Smoking
E. None of these
Correct Ans B: Increased blood pressure

88. Bronchitis is the abnormality of

M
A. Liver
B. Respiratory tract

O
C. Kidney

C
D. Blood
E. None of these
Correct Ans B: Respiratory tract S.
ER

89. Gonorrhea can be cured with


IV

A. Penicillin
G

B. Rifampicin
C. Antitoxin
BS

D. Biocillin
E. None of these
JO

Correct Ans A:Penicillin

90. The multidrug treatment against leprosy includes

A. Antitoxin
B. Penicillin
C. Rifampicin
D. Biocillin
E. None of these
Correct Ans C:Rifampicin
91. Hansen's disease is

A. Plague
B. Tetanus
C. Tuberculosis
D. Leprosy
E. None of these
Correct Ans D:Leprosy

92. Physical dependence is the result of

M
A. Alcoholism
B. AIDS

O
C. Drugs

C
D. Polygamy
E. None of these
Correct Ans A:Alcoholism
S.
ER

93. Which one of the following is a sexually transmitted disease?


IV

A. Leprosy
G

B. Syphilis
C. Tuberculosis
BS

D. Tetanus
E. None of these
JO

Correct Ans B: Syphilis

94. Malaria is spread by

A. Air
B. Culex
C. Female Anopheles mosquito
D. Male Anopheles
E. None of these
Correct Ans C: Female Anopheles mosquito
95. The painful swelling in either one or both of the parotid gland is due to

A. Tetanus
B. Chicken pox
C. Mumps
D. Measles
E. None of these
Correct Ans C:Mumps

96. The important anti-tuberculosis drug is

M
A. Aureomycin
B. Streptomycin

O
C. Chloromycin

C
D. Neomycin
E. None of these
Correct Ans B:Streptomycin
S.
ER

97. he effective live vaccine available for the control of mumps is


IV

A. MMR
G

B. DT
BS

C. BCG
D. DPT
E. None of these
JO

Correct Ans A:MMR

98. The antiviral drug used in the "control of influenza is"

A. Biocillin
B. Ampicillin
C. Amantadine
D. Septran
E. None of these
Correct Ans C: Amantadine
99. The symptoms of measles are

A. fever, cough, sneezing and redness of eyes


B. emption of small red spots
C. inflammation of mucous membrane of the nose
D. All of the above
E. None of these
Correct Ans B: emption of small red spots

100. Measles is caused by - virus.

A. Monili virus
B. HAV virus

M
C. Bacteriophage

O
D. HIV virus
E. None of these

C
Correct Ans A: Monili virus
S.
ER
101. The vitamin given for the treatment of common cold is,

A. vitamin 'A’
IV

B. vitamin 'B‘
C. vitamin 'C'
G

D. vitamin 'D'
BS

E. None of these
Correct Ans C: vitamin 'C'
JO

102. ”Hepatitis" means

A. liver ailment
B. lung ailment
C. stomach ailment
D. kidney ailment
E. None of these
Correct Ans A: liver ailment
103. Prophylaxis means

A. Preventive treatment against diseases


B. Immunization
C. Breaking the routes of transfer of diseases
D. Non-specific measures
E. None of these
Correct Ans A: Preventive treatment against diseases

104. Diseases spread by articles like cups, spoons are called as

A. Dust homes

M
B. Vehicle borne

O
C. Air borne diseases

C
D. Fomite borne diseases
E. None of these
S.
Correct Ans D: Fomite borne diseases
ER

105. Which one of the following is transmitted by placenta?


IV

A. Tetanus
B. Diphtheria
G

C. Rabies
BS

D. Syphilis
E. None of these
Correct Ans D: Syphilis
JO

106. Whooping enough is transmitted by

A. Droplet infection
B. Bite of animal
C. Direct contact
D. None of these
E. All a,b,c
Correct Ans C: Droplet infection
107. The disease and infections which are naturally transmitted between vertebrate
animal and man are

A. Communicable diseases
B. Non-communicable diseases
C. Contact diseases
D. Zoonoses
E. None of these
Correct Ans C: Contact diseases

108. The vaccine given to newborn child is

A. BCG

M
B. TT

O
C. DT
D. MMR

C
E. None of these
Correct Ans B: TT S.
ER

109. The kind of immunity developed in a person who-had an attack of chickenpox is


IV

A. natural passive acquired immunity


G

B. artificial passive acquired immunity


C. natural active acquired immunity
BS

D. artificial active acquired immunity


E. None of these
JO

Correct Ans C: natural active acquired immunity

110. Inoculation of vaccines to prevent diseases is

A. Prepathogenesis
B. Immunisation
C. Incubation
D. Hibernation
E. None of these
Correct Ans B: Immunisation
111. Consider the following statements:

1. Copper 'T' is made of Copper and Plastic

2. It prevents fertilisation

3. It remains for a period of 3 years

4. It is placed in the Uterus

A. 1 and 2 are correct


B. 3 and 4 are correct
C. 4 alone is correct
D. All are correct

M
E. None of these

O
Correct Ans D: All are correct

C
112. AIDS is caused by S.
ER
A. Bacteria
B. Protozoa
C. Helminth
IV

D. HIV Virus
G

E. None of these
Correct Ans D: HIV Virus
BS
JO

Question 113
An 89 year old man has a basal cell carcinoma on his forehead which requires excision. He has
dementia. The clinic nurse feels he is not competent to give consent for surgery.

Which is the most appropriate action for obtaining consent?

A. Ask a psychiatrist to assess his cognitive function


B. Ask his general practitioner to sign the consent form
C. Ask his wife to sign the consent form
D. Ask the patient to sign the consent form
E. Ask the surgeon to assess his mental capacity

ANSWER

E. Ask the surgeon to assess his mental capacity

Question 114

M
A 21 year old woman has increasing severe pain in her left leg. She had an undisplaced fracture

O
of the midshaft of her left tibia 2 days ago which was treated conservatively with a plaster cast.

C
Her toes are warm and pulses are present.

Which is the most important immediate action?


S.
ER
A. Elevate the limb
B. Refer to orthopaedics
C. Remove the plaster cast
D. Replace the cast with increased padding
IV

E. Re-X-ray the fracture site


G

ANSWER
BS

C. Remove the plaster cast


JO

Question 115
A 14 year old girl attends the GP surgery requesting the oral contraceptive pill (OCP). She is
sexually active and her boyfriend is in the year above her at school. Her parents are unaware of
the appointment and her request for OCP.

Which is the most appropriate management?

A. Advise her about safe sex and prescribe the OCP


B. Contact her parents
C. Contact the local safeguarding team
D. Contact the police
E. Explain it is illegal to prescribe the OCP for her

ANSWER

A. Advise her about safe sex and prescribe the OCP

Question 116
A 22 year old man has a painful right eye for 1 day. He had similar symptoms 1 year ago. He has

M
had episodic back pain and stiffness for 4 years which is relieved by exercise and ibuprofen.

O
His right eye is red and his vision is blurred.

C
Which is the most likely cause of his red eye?

A.
B.
Chorioretinitis
Conjunctivitis
S.
ER
C. Episcleritis
D. Keratitis
E. Uveitis
IV

ANSWER
G

E. Uveitis
BS
JO

Question 117
A healthy boy is born by vaginal delivery at 40 weeks gestation. His mother had confirmed acute
hepatitis B during this pregnancy.

Which preventative intervention should be given immediately to the baby?

A. Confirm hepatitis B status of baby as first step


B. Full course of hepatitis B vaccine
C. Full course of hepatitis B vaccine and hepatitis B immunoglobulin
D. Hepatitis B immunoglobulin
E. Hepatitis B vaccine as a single dose
F.

ANSWER

C. Full course of hepatitis B vaccine and hepatitis B immunoglobulin

Question 118
A 66 year old man has tiredness and low back pain for 2 months. He has type 2 diabetes and
hypertension and takes metformin and ramipril. He is a non-smoker and does not drink alcohol.
His pulse rate is 88 bpm and BP 138/82mmHg. Clinical examination is normal.

M
Investigations:
Haemoglobin 78 g/L (130–175)

O
White cell count 3.4 × 109/L (3.0–10.0)
Platelet count 80 × 109/L (150–400)

C
Corrected calcium 2.71 mmol/L (2.2–2.6)
Albumin 36 g/L (35–50)
S.
X-ray of lumbar spine: generalised osteopenia and vertebral collapse at multiple levels.
ER
Which is the most appropriate next investigation?
IV

A. Serum angiotensin converting enzyme


B. Serum parathyroid hormone
C. Serum prostate specific antigen
G

D. Serum protein electrophoresis


BS

E. Serum vitamin D

ANSWER
JO

D. Serum protein electrophoresis

Question 119
A 17 year old woman has a painless lump on her right breast for 4 weeks. There is no history of
trauma. She takes the oral contraceptive pill. Her maternal grandmother had breast cancer.

She has a 1 cm painless lump in the upper outer quadrant of her right breast.

Which is the most appropriate initial investigation?


A. Excision biopsy
B. Fine needle aspiration cytology
C. Magnetic resonance imaging
D. Mammography
E. Ultrasound scan of the breast

ANSWER

E. Ultrasound of the breast

Question 120

M
A 40 year old man takes an overdose of his antidepressant tablets and is seen in the Emergency
Department. He is medically fit for discharge. He has bipolar affective disorder. He is tearful and

O
feels hopeless about his future.

C
Which is the most appropriate next management step?

A.
B.
Arrange liaison psychiatric assessment
Arrange psychiatric outpatient follow-up
S.
ER
C. Discharge to the care of his general practitioner
D. Referral to clinical psychologist
E. Referral to local alcohol treatment team
IV

ANSWER
G

A. Arrange liaison psychiatric assessment


BS
JO

Question 121
A 6 week old girl has bronchiolitis and is seen in the Emergency Department. She is medically
stable and ready for discharge. There were no problems in pregnancy. She was born at term by
normal vaginal delivery. She is an only child. Her mother says that she cannot cope with her new
baby.

Which is the most appropriate initial management?


A. Arrange temporary foster care
B. Contact social services
C. Refer back to her GP
D. Refer her to safeguarding team
E. Organise a Place of Safety order

ANSWER

B. Contact social services

Question 122

M
A 4 week old boy has episodes of vomiting since birth. The vomiting occurs soon after feeding.

O
He does not vomit after every feed and it is not related to the amount he takes. He is otherwise

C
well and is gaining weight appropriately.

Which is the most likely diagnosis?


S.
ER
A. Cows milk protein allergy
B. Gastro-oesophageal reflux
IV

C. Lactose intolerance
D. Pyloric stenosis
G

E. Urinary tract infection


BS

ANSWER

B. Gastro-oesophageal reflux
JO

Question 123
A 19 year old woman has worsening vaginal discharge and bilateral iliac fossa pain for 3 days.
Her vaginal discharge is foul smelling and she has dyspareunia. She is feverish and feels unwell.

Her temperature is 39.0°C.

Which combination of organisms are most likely to cause these symptoms?


A. Candida albicans and Gardnerella vaginalis
B. Chlamydia trachomatis and Candida albicans
C. Neisseria gonorrhoeae and Chlamydia trachomatis
D. Neisseria gonorrhoeae and Gardnerella vaginalis
E. Trichomonas vaginalis and Candida albicans

ANSWER

C. Neisseria gonorrhoeae and Chlamydia trachomatis

Question 124

M
An 85 year old man has a fall at home and is seen in the Emergency Department. He had
incontinence of urine and had been lying on the floor all night. He had a myocardial infarction 8

O
years ago. He takes aspirin, atorvastatin and ramipril.
His pulse rate is 94 bpm and BP 106/76 mmHg.

C
Investigations:
• White cell count 14.3 × 10⁹/L (3.0–11.0)
• Urea 30.4 mmol/L (2.5–7.8)
• Creatinine 158 micromol/L (60–120)
• Creatinine kinase 11025 U/L (25–200)
S.
ER
ECG: Q waves in lead III.
Which is the most likely cause of his raised creatinine kinase?
IV

A. Acute kidney injury


G

B. Acute myocardial infarction


C. Dermatomyositis
BS

D. Rhabdomyolysis
E. Urinary tract infection

ANSWER
JO

D. Rhabdomyolysis

Question 125
A 30 year old man has had weakness in his left arm and leg for three days. He is HIV positive
with a CD4 count of 50 cells/mm3. He is not taking anti-retroviral therapy.

His temperature is 37.5oC, pulse rate 95 bpm and BP 140/90 mmHg. He has power of 3/5 in his
left arm and 4/5 in his left leg there is no other focal neurological signs.
Investigations:
CD4 count 50 cells/mm3
CT scan of brain: multiple ring enhancing lesions with surrounding oedema.

What is the most appropriate antimicrobial therapy to prescribe?

A. Amphotericin and ceftazidime


B. Dexamethasone and acyclovir
C. Ganciclovir and ceftazidime
D. Isoniazid and rifampicin
E. Pyrimethamine and sulphadiazine

ANSWER

E. Pyrimethamine and sulphadiazine

M
O
C
Question 126
S.
A 13 year old boy’s mother is concerned that he has no friends and she brings him to the GP
ER
surgery. He does not play with other children. He likes model cars and has 1,000 cars in his
collection. He is indifferent to either praise or criticism from his mother or teachers.

Which is the most likely diagnosis?


IV

A. Attachment disorder
G

B. Attention deficit hyperactivity disorder


C. Autistic spectrum disorder
BS

D. Obsessive compulsive disorder


E. Schizoid personality disorder

ANSWER
JO

C. Autistic spectrum disorder

Question 127
An 19 year old woman is worried that she is overweight and is seen in the GP surgery. She has
reduced her food intake and has been exercising for 2 hours each day for 18 months. She has
amenorrhoea. She is in her first year of university and is struggling with her studies.

Her body mass index is 15.5 kg/m2 and her blood pressure is 90/60 mmHg.
Which specialist service should she initially be referred to?

A. Acute medical services


B. Dietician
C. Eating disorder service
D. Psychodynamic services
E. University student counselling services

ANSWER

C. Eating disorder service

M
O
Question 128

C
A 28 year old woman requests contraception from the GP surgery. She has a 28 day menstrual
S.
cycle with a 6 day very heavy menstrual bleed. She had a pulmonary embolism in her previous
pregnancy.
ER
Which is the most appropriate contraception?

A. Combined oral contraceptive pill


IV

B. Copper intrauterine contraceptive device


C. Levonorgestrel-releasing intrauterine system (Mirena coil)
G

D. Progestogen implant
E. Progestogen-only pill
BS

ANSWER

C. Levonorgestrel-releasing intrauterine system (Mirena coil)


JO

Question 129
A 65 year old man has pain in his abdomen and is seen on the surgical inpatient ward. He had a
closure of his colostomy 5 days ago.

His temperature is 36.5°C, pulse rate is 76 bpm, BP 124/82 mmHg and respiratory rate is 15
breaths per minute. There is a tender, localised fluctuant swelling 4 cm in diameter in the wound.
Which is the most appropriate management?

A. Abdominal support
B. Antibiotics
C. Laparotomy
D. Local exploration of wound
E. Observation

ANSWER

D. Local exploration of wound

Question 130

M
10,000 women underwent cervical cytology. 1000 had abnormal cells and had a second smear

O
test. 100 of these women were confirmed as having pre-malignant cells and underwent laser
ablation. Ten women who were not identified as needing a second cervical smear developed

C
cervical cancer within 6-12 months.

S.
Which term describes the group of 10 women who were not identified as needing a second smear
but developed cervical cancer within 6-12 months?
ER
A. Case control group
B. False negative group
IV

C. Incident group
D. True negative group
E. True positive group
G
BS

ANSWER

B. False negative group


JO

Question 131
A 38 year old man is constantly worried about his house being infected by germs for 3 months
and he is seen in the GP surgery. He is worried about his personal safety and has been checking
that the doors and windows are locked 20-30 times a day for 8 weeks. He has been washing his
hands every time he touches the locks.
Which is the most appropriate initial management?

A. Antidepressant
B. Antipsychotic
C. Anxiolytic
D. Cognitive behaviour therapy
E. Psychodynamic psychotherapy

ANSWER

D. Cognitive behaviour therapy

M
Question 132

O
C
An 8 year old girl has a frequent cough at night and mild exercise-induced wheeze for 3 months.
She has asthma and takes low-dose inhaled corticosteroids twice daily and a short-acting

S.
bronchodilator as required. She takes her treatment as recommended. Her inhaler technique is
good.
ER
Which is the most appropriate next step in her treatment?

A. Add leukotriene receptor antagonist


IV

B. Add oral modified release theophylline


C. Add short course of oral corticosteroid
G

D. Increase dose of inhaled corticosteroid


E. Increase frequency of short acting bronchodilator
BS

ANSWER

A. Add leukotriene receptor antagonist


JO

Question 133
An 84 year old woman has worsening confusion. She has visual hallucinations, disturbed sleep
and urinary incontinence for 1 week. She has Alzheimer’s dementia and arthritis and has recently
started taking paracetamol.

Her abbreviated mental test score is 2/10.


Which is the most likely underlying reason for her deterioration?

A. Depression
B. Encephalitis
C. Medication related confusion
D. Progression of dementia
E. Urinary tract infection

ANSWER

E. Urinary tract infection

Question 134

M
A 38 year old man has worsening vertigo and tremor. His symptoms are more pronounced in the

O
morning. He has longstanding alcohol dependence and drinks 60 units of alcohol per week.

C
His abbreviated mental test score is 9/10. He has an intention tremor, horizontal nystagmus and
walks with a broad-based gait.

Which is the most likely diagnosis?


S.
ER
A. Acute alcohol withdrawal syndrome
B. Benign positional vertigo
IV

C. Cerebellar degeneration
D. Korsakoff syndrome
G

E. Wernicke encephalopathy
BS

ANSWER

C. Cerebellar degeneration
JO

Question 135
A 32 year old woman has a febrile illness and swelling of the small joints of her hands for 2
days. She is seen in the GP surgery. She has not travelled outside the UK. She has two young
children.

She has a maculopapular rash and palpable, small cervical lymph nodes. The small joints of her
feet, wrists and knees are swollen.
Which is the most likely diagnosis?

A. Psoriasis
B. Reactive arthritis
C. Rheumatoid arthritis
D. Sarcoidosis
E. Systemic lupus erythematosus

ANSWER

B. Reactive arthritis

M
Question 136

O
A 78 year old woman has pain in her left thigh and calf on walking for 8 weeks. The pain is
relieved with rest. She has a 40 pack-year smoking history.

C
Her left popliteal and pedal pulses are not palpable.

Investigation:
S.
ER
Hand held Doppler: monophasic waveform in her left Dorsalis Pedis artery.

Which is the most likely site of arterial pathology?


IV

A. Anterior Tibial
B. Dorsalis Pedis
G

C. External Iliac
D. Internal Iliac
BS

E. Profunda Femoris

ANSWER
JO

C. External Iliac

Question 137
A 53 year old woman has anorexia and increasing breathlessness. She has stage 4
adenocarcinoma of the lung and now requires full support with her activities of daily living. She
takes diazepam, cyclizine and zopiclone.
Investigations:
• Sodium 122 mmol/L (135-146)
• Potassium 3.2 mmol/L (3.5-5.3)
• Urea 6.2 mmol/L (2.5-7.8)
• Creatinine 58 mmol/L (45-95)

Which is the most appropriate management of her electrolyte disturbance?

A. 1.8% sodium chloride by intravenous infusion


B. DDAVP nasally
C. Demeclocycline orally
D. Fluid restriction
E. No treatment indicated

ANSWER

M
E. No treatment indicated

O
C
Question 138
S.
A 3 day old baby boy is noted to have ambiguous genitalia at his postnatal check.
Investigations:
ER
• Sodium 125 mmol/L (133-146)
• Potassium 5.2 mmol/L (3.4–6.0)
• Urea 5.0 mmol/L (0.8-5.5)
IV

• Creatinine 70 micromol/L (21–75)/li>


• Bicarbonate 22 mmol/L. (22-29)
What is the most likely diagnosis:
G
BS

A. Acute renal failure


B. Congenital adrenal hyperplasia
C. Diabetes insipidus
D. Hyponatraemic dehydration
JO

E. Renal tubular acidosis

ANSWER
B. Congenital adrenal hyperplasia

Question 139
A 2 year old boy hurt his left arm after a fall from his tricycle. He stood up and started to cry but
then went pale, unconscious and rigid. He recovered after 1-2 minutes. He had a similar episode
3 months ago after falling down some steps.
Examination is normal.

Which is the most appropriate investigation?

A. CT scan of the head


B. Electroencephalogram
C. Full blood count
D. No investigation indicated
E. Skeletal survey

ANSWER

D. No investigation indicated

M
Question 140

O
A 16 year old girl has an enlarging lump on the right side of her neck for 6 weeks. She is seen in

C
the GP surgery. She has no other symptoms.

S.
She has a 2 x 2 cm lymph node in the right anterior triangle of the neck and several smaller
cervical lymph nodes. Her tonsils are enlarged and covered by a grey membrane.
ER
Which is the most likely diagnosis?
IV

A. Infectious mononucleosis
B. Leukaemia
G

C. Lymphoma
D. Sarcoidosis
BS

E. Tuberculosis

ANSWER
JO

A. Infectious mononucleosis

Question 142
A 66 year old man has increasing abdominal pain. He had an anterior resection 5 days ago.
His temperature is 38.5 ˚C, pulse rate 120 bpm and BP 90/60 mmHg. His abdomen is tender and
there is generalised rigidity.

Which is the most likely diagnosis?

A. Hypovolaemia
B. Leaking anastomosis
C. Secondary haemorrhage
D. Urinary tract infection
E. Wound infection

ANSWER

B. Leaking anastomosis

M
O
C
Question 142
S.
An 83 year old woman has visual hallucinations and is confused for 4 hours. She is on the
ER
medical inpatient ward and is being treated for pneumonia. She is restless and verbally abusive
towards the staff who she believes are trying to poison her. She has no previous psychiatric
history.
IV

Which is the most likely diagnosis?


G

A. Delirium
B. Delusional disorder
BS

C. Charles Bonnet syndrome


D. Parkinsons disease dementia
E. Psychotic depression
JO

ANSWER

A. Delirium
SESSION

SET 1 ENGLISH LANGUAGE


1. Which of the following phrases A, B, C and D given below in the statement should
replace the phrase printed in bold in the sentence to make it grammatically correct?
If the sentence is correct as it is given and ‘No Correction is required’, mark E as the
answer.

M
Since their son’s demise last week, the old couple were been in shock.

O
C
A. were
B. will be
C.
D.
have been
was being
S.
ER
E. No correction
IV

2. Choose an appropriate word from the options to suitably fill the blank in the
sentence below so that the sentence makes sense, both grammatically and
G

contextually.
BS

You your report by this time next week.


JO

A. have finished
B. has finished
C. will have finished
D. would have finished
E. have been finished

3. Fill in the blank with the suitable article:

sugar is bad for your health.

A. Some
B. a
C. an
D. the
E. No article

4. Fill in the blank with the suitable article:

The government is planning to open university in the city.

M
O
C
S.
ER
IV
G
BS
JO
A. some
B. a
C. an
D. the
E. No article is required
5. Which of the phrases A, B, C and D given below should replace the phrase given in
bold in the following sentence to make the sentence grammatically correct? If the
sentence is correct, as it is and ‘No correction is required’, mark E as the answer.

Radha’s three children, Shantana, Manu and Meera are talented, but the latter excels

over the other two.

M
A. the last excels

O
B. latter excel
C. the latter excelling

C
D. the last excelling
No correction required
S.
ER
6. Which of the following phrases (A), (B), (C), (D) given below in the statement
should replace the phrase printed in bold in the sentence to make it grammatically
correct? If the sentence is correct as it is given and ‘No Correction is required’, mark
IV

(E) as the answer.


G

Our neighbours are always trying to keep up with our lifestyle.


BS

A. to keep on
B. to keep at
JO

C. to keeping on
D. to keep off
E. No Correction is required

7. Select the correct tense.

Kiran had been working in a bank for some years.

A. Simple past
B. Past continuous
C. Past perfect continuous
D. Past perfect.
E. None of the above

M
O
C
S.
ER
IV
G
BS
JO
8. Select the correct plural form for the given word:

Thief

A. Thiefs
B. Thief
C. Thieves
D. Thievs
E. None of the above

M
O
9. Select the correct plural form for the given word:

C
Roof
S.
ER
A. Roofs
B. Roof
IV

C. Roofes
D. Roovs
E. None of the above
G
BS

10. Pick out the most effective words from the given words to fill in the blank to make
JO

the sentence meaningfully complete.

I with my sister I find an apartment.

A. am living, until
B. have been living, since
C. had lived, since
D. had been living, since
E. am living, since

11. Which of the phrases A, B, C and D given below should replace the phrase given in
bold in the following sentence to make the sentence grammatically correct? If the
sentence is correct, as it is and ‘No correction is required’, mark 5 as the answer.
We must treat a statement as a rumour until they are confirmed with proof.

A. till they are confirmed


B. until they are confirming
C. until it is confirmed
D. until it is confirming
E. No correction required

M
O
C
S.
ER
IV
G
BS
JO
12. ich of the phrases 1, 2, 3 and 4 given below should replace the phrase given in
bold in the following sentence to make the sentence grammatically correct? If the
sentence is correct, as it is and ‘No correction is required’, mark 5 as the answer.

The officer appreciated his subordinate’s many attempt to bravely confront the
miscreants.

A. many attempting brave confronts


B. many brave attempts to confront
C. repeated attempts to brave confront
D. many attempts of brave confront

M
E. No correction required

O
C
S.
13. Which of the phrases 1, 2, 3 and 4 given below should replace the phrase given in
bold in the following sentence to make the sentence grammatically correct? If the
ER
sentence is correct, as it is and ‘No correction is required’, mark 5 as the answer.

We admire his attempting to climb the summit in such bad weather.


IV
G

A. his attempting to climb


B. his attempt of climbing
BS

C. him for attempt of climb


D. his for attempt to climbing
E. No correction required
JO

14. Fill in the blank with suitable conjunction

He will never pass, hard he may try.

A. However
B. Whatever
C. Never
D. Whenever
E. Whichever
15. Form an adjective from the given word

Gold

A. The Gold
B. Golder
C. Golden

M
O
C
S.
ER
IV
G
BS
JO
D. Goldest
E. None

16. Select the one which expresses the same sentence in the passive voice.

John has been trying to get a new job for two years.

A. Getting a new job is being tried by John for two years.


B. Getting a new job is tried by John for two years.
C. For two years getting a new job has been tried by John.

M
D. For two years, John has tried to get a job.
E. None of these

O
C
S.
17. Which of the following phrases (A), (B), (C), (D) given below in the statement
should replace the phrase printed in bold in the sentence to make it grammatically
ER
correct? If the sentence is correct as it is given and ‘No Correction is required’, mark
(E) as the answer.
IV

The doctor told him to cut on his daily sugar consumption.


G

A. to cut down on
BS

B. to cutting on
C. to caught on
D. to catch on
JO

E. No Correction is required

18. The sentence below has been broken up into four parts sequentially. Choose the
part which has a mistake or word(s) is (are) not quite appropriate. If there are no
errors, mark 5 as the answer.

Advocates of non-violence (1)/ have always adviced (2)/ peaceful solutions (3)/ to
the most complex problems (4)./ No error(5)

A. 1
B. 2
C. 3
D. 4
E. 5

JO
BS
G
IV
ER
S.
C
O
M
19. The sentence below has been broken up into four parts sequentially. Choose the
part which has a mistake or word(s) is (are) not quite appropriate. If there are no
errors, mark 5 as the answer.

The farmer (1)/ felt badly about (2)/ losing all of his crops (3)/ overnight (4)./ No
error(5).

A. 1
B. 2
C. 3
D. 4
E. 5

M
O
C
20. Fill in the blank with the correct form of tense:

S.
He a fortune by writing various kinds of books.
ER
A. Did making
B. Would made
C. Is making
IV

D. Will made
E. Will make
G
BS
JO
ANSWERS
1. C 2. E 3. E 4. B 5. A

6. E 7. C 8. C 9. A 10. A

11. C 12. B 13. B 14. A 15. C

16. E 17. A 18. B 19. B 20. A

M
O
EXPLANATIONS

C
S.
1. From the sentence the tense seems most likely to be present perfect continuous.
Hence, the words ‘have been’ are most suitable.
ER
Solution: C
IV
G
BS

2. The sentence is in future perfect tense. This can be understood by the words ‘by
this time next week’. Therefore, the future perfect form of the verb ‘will finish’ has to
be used. The answer is ‘will have finished’.
JO

Solution: C

3. There are some instances when we omit the use of articles. So when we are
referring to substances or abstract nouns i.e. uncountable nouns in the general
sense, there is no need to use any article. E.g. Gold is precious metal, honesty is the
best policy.

Solution: E

4. The word ‘university’ begins with a consonant sound ‘yu’ and therefore the correct
article is ‘a’ and not ‘an’, even though the word begins with a vowel.

Solution: B
5. When three people are there, ‘last’ is used instead of latter to describe the third.
Hence, the ‘last’ is correct. Also, ‘excelling’ cannot be used as it is used for an
ongoing activity.

Solution: A

M
O
C
S.
ER
IV
G
BS
JO
6. 'Keep up' (with someone/something) means to stay at the same level as someone
or something and fits the context of the sentence. Also none of ‘keep on’ to continue
doing something, ‘keep at’ means to persevere and ‘keep off’ means avoid
encroaching on or touching, is implied in the sentence.

Solution: E

7. (Had + been + root + ing) is used to make past perfect continuous tense and

M
indicates that an activity was being carried on in the past, continued for some time
and is no longer continued.

O
C
Solution: C

S.
ER

8. The plural of ‘thief’ is ‘thieves’. One cannot always apply the rule of adding an ‘s’ to
IV

change singular to plural. For words ending with f, the rule is usually to change the f
to- ves.
G
BS

Solution: C
JO

9. The plural of ‘roof’ is ‘roofs’. In such cases, adding an ‘s’ converts singular to plural.
This is an exception to the rule that a plural is formed by adding -ves to words ending
with a f.

Solution: A

10. The first blank will have the present continuous tense as it is for an action that is
happening around now, and is not permanent or habitual. Present perfect continuous
tense would have been used if the action began in the past and is continuing now
(eg. I have been living with my sister since last week). The structure of forming
present continuous tense is am/are/is + verb + ing, so the correct option will be ‘am
living’. ‘Until’ is to be used in the second blank and not ‘since’ as ‘since’ refers to
something that has happened in a time span measured from a certain point in time
(eg. I have not found an apartment since last year). So, all options with ‘since’ are
incorrect. ‘Until’ should be used in the second blank as it means the action will
happen in the future.

M
O
C
S.
ER
IV
G
BS
JO
Solution: A

11. ‘A statement’ is used which is singular. Hence, the plural form ‘they’ must be
replace by ‘it’. The statement is not performing the action of ‘confirmation’. Hence, ‘it
is confirmed’ is more appropriate.

Solution: C

M
12. If ‘many’ or ‘repeated’ is used, ‘attempts’ must be there since it is plural. ‘To

O
confront’ is correct as the answer ‘miscreants’ to the question ‘whom to confront?’ is
given. The adjective ‘brave’ should describe the attempts made by the subordinate

C
and is thus placed before attempts.

Solution: B S.
ER
IV
G

13. ‘Attempting to climb’ is incorrect as we do not know if the action of ‘attempting’ is


going on. ‘For’ cannot be used before the word attempt. ‘Of climbing’ is correct and
BS

we cannot use ‘of climb’.

Solution: B
JO

14. ‘Never’ is an adverb and not a conjunction and thus can be eliminated. ‘Whatever’,
‘whichever’ and ‘whenever’ need to be followed by a noun and pronoun which is not
the case here. ‘However’ is a conjunctive adverb which joins two independent clauses
which is the case here. It is being used to contrast the situation.

Solution: A

15. Adjective is something that describes a noun. Eg: Golden Sun

Solution: C
16. There are some tenses which can’t be converted into passive voice. Following is
the list

Present perfect continuous tense

Past perfect continuous tense

Future continuous tense

M
O
C
S.
ER
IV
G
BS
JO
Future perfect continuous tense

The given sentence is in present perfect continuous and therefore does not have a
passive form.

Solution: E

17. ‘Cut down on’ means to reduce in number or size. The sentence talks about a

M
doctor and a person’s sugar consumption and therefore the expression perfectly fits

O
the context

C
Solution: A

S.
ER
IV

18. The use of “adviced” is incorrect. ‘Advice’ is a noun meaning a suggestion or


beneficial course of action, and cannot be in past tense. ‘Advised’ should be used as
G

‘advise’ is a verb which means to give advice.


BS

Solution: B
JO

19. ‘Badly’ is used incorrectly in the sentence. ‘Badly’ is an adverb which can mean
several things-

In an unsatisfactory manner – e.g. The Indian cricket team played badly in the finals.

In an unfavourable manner – e.g. Do not think too badly of me.

In an unacceptable manner – e.g. The student was punished because he was


behaving badly.

Intensely – e.g. I want to go to Paris so badly.


To feel guilty – e.g. I felt badly about my outburst.

The word ‘bad’ on the other hand can be used as an adjective. Notice that in this
sentence, the word ‘badly’ is being used to describe the state of the farmer. So an
adjective is to be used. Hence the word ‘badly’ is to be replaced with ‘bad’ to make
the sentence correct.

Solution: B

M
O
C
S.
ER
IV
G
BS
JO
20. ‘Writing’ signifies that he is still writing books and that shows that he is still making
fortune as both the tenses will be same so, the first part will also be in present
continuous tense. Hence, the correct answer will be ‘is making’ in the sentence.

Solution: C

M
O
C
S.
ER
IV
G
BS
JO
MATHEMATICS

1. The inequality equation given is y < 3 which of the following diagrams explains the
equation?

A.

M
B.

O
C
C.
S.
ER
IV
G

D.
BS
JO

2. The product of two consecutive positive even numbers is 288. By constructing a


quadratic equation and solving it, find the two numbers.

A. 14

B. 20

C. 7

D. 10
3. If Boneri adds 2 to the numerator of a fraction, the fraction becomes 1⁄3. If he
subtracts 3 from the denominator of the fraction, it becomes 1⁄4. What is the fraction?

A. 1⁄5

B. 2⁄5

C. 3⁄5

M
O
C
S.
ER
IV
G
BS
JO
D. 4⁄5

4. Given that P = {x: 1 ≤ x ≤ 6} and Q = {x: 2 ˂ x ˂ 10}, where x is an integer. Find n (P ∩


Q).

A. 4

M
B. 6

O
C. 8

C
D. 10

S.
ER

5. Solve the equation [x + 3][x + 1]>9 + x2.


IV

A. x < 11/2
G

B. x 11/2
BS

C. x > 11/2

D. x 11/2
JO

6. Find the number of term of the sequence that will make up 240. The given
sequence are 13, 16, 19, 22, ...............

A. 9

B. 17

C. 27

D. 29
7. If the second and fourth terms of a G.P. are 8 and 32 respectively, what is the sum
of the first four terms?
A. 28

B. 40

C. 48

D. 60

M
O
C
S.
ER
IV
G
BS
JO
8. Calculate the sum of the tenth terms of the G.P. 5, -21⁄2, 11⁄4 ...............

A. 4.77

B. 3.34

C. 6.852

D. 11.27

M
O
C
find the common difference.
A. 6.31
S.
9. If the 8th term of an A.P. is twice the 4th term and the sum of the first 5 terms is 47,
ER

B. 3.13
IV

C. 4.42
G
BS

D. 2.24

10. The 4th term of an A. P. is 13 while the 10th term is 31. Find the 21st term.
JO

A. 175.

B. 85.

C. 64.

D. 45.

11. One third of a number y is subtracted from 5 and the result is at most 3. What is
the range of values of y?

A. y > 6

B. y ≥ 6
C. y ≤ 6

D. y < -6

M
O
C
S.
ER
IV
G
BS
JO
12. From the given diagram drawn, what is the inequality equation?

A. x > -3

M
B. x 3

O
C. x > 3

C
D. x -3
S.
13. The perimeter of a rectangle is 42cm and its area is 68cm2. Find its length and
ER
breadth.
IV

A. 17cm ; 3cm
G

B. 14cm ; 3cm
BS

C. 17cm ; 4cm

D. 21cm ; 2cm
JO

14. Complete the table giving the values for the relation y = 2x2 + x - 7, find the value
of A.

A. 3

B. 14
C. -1

D. 6

15. Given that T = {x: -2 ˂ x ≤ 9} where x is an integer. What is n (T)?

A. 9

M
O
C
S.
ER
IV
G
BS
JO
B. 10

C. 11

D. 12

16. The first and last term of an A.P. are 10 and 90, if the sum of term is 750, the
number of terms is used to find the common difference if the sum is 1400.

A. 11.90

M
B. 10.90

O
C. 12.90

C
D. 5.7
S.
ER
IV

17. Find the sum of 7th term of the G.P.


G

12, 6, 3. Find the sum to infinity.


BS

A. 14
JO

B. 16

C. 40

D. 24

18. Find the nth term of the given progression 36, 12, 4,.............
A. 14n + 1
B. 12n - 1

C. 12n + 1

D. 14n -1

19. The twelfth term of an arithmetical progression is twice the sixth term. The first
term is equal to
A. the common difference.

B. half of the common difference.

M
O
C
S.
ER
IV
G
BS
JO
C. zero.

D. double the common difference.

20. Find the sum of 22 terms of an A.P. 4, 6, 8.


A. 550

M
B. 650

O
C
C. 750

D. 450 S.
ER
IV
G
BS
JO
ANSWERS
1.c 2. A 3. A 4. A 5.C
6.
7.D 8.B 9.B 10. A
A
11.C 12.C 13.C 14.C 15.C

16.A 17.D 18.B 19.A 20.A

M
O
C
S.
EXPLANATION
ER
1. The correct answer is option [C]. Solution:
IV

The broken lines = < or > while the unbroken lines = or


G

The unshaded portion is the required region.


BS
JO

2. The correct answer is option [A]. Solution: [x + 2][x + 4] = 288; x2 + 6x + 8 = 288; x2 +


6x - 280 = 0; x[x + 20] - 14[x + 20] = 0 → [x - 14][x + 20] = 0, therefore, x = 14 or -20.
The number is 14.

3. The correct answer is option [A]. Solution: Let the fraction be /y. x + 2/y = 1⁄3;
x

3[x + 2] = y; 3x + 6 = y ----- [i];


x
/y - 3 = 1/4; 4x = y - 3 ------ [ii].

Substitute y in equation [i] into equation [ii];


4x = 3x + 6 - 3; 4x - 3x = 3,

therefore, x = 3; y = 3x + 6 y = 3[3] + 6 = 9 + 6 = 15.

The fraction x/y = 3/15 = 1/5.

M
O
C
S.
ER
IV
G
BS
JO
4. The correct answer is option [A].

5. The correct answer is option [C].

Solution: [x + 3][x = 1] > 9 + x2.

Expand

x2 + 4x + 3 > 9 + x2 4x > 9 - 3 x > 6/4.

M
O
Therefore, x >11/2.

C
S.
6. The correct answer is option [A]. Solution: Hint [The question forms A.P.]. Using the
equation Sn = [n 2a + (n - 1) d]/2, where a = 13, d = 3, Sn = 240. Solve using the formula
ER
given.
IV
G

7. The correct answer is option [D].


BS

The general form of a G.P. is a, ar, ar2,...., arn - 1 (for n terms).


Here ar =second term = 8; ar3 = fourth term = 32. [ar3/ar = r2]
JO

32
/8 = 4 = r2 (r = (4 = 2 (where r=common ratio, a = 1st term).
For ar = 8
2a = 8
a=4
For r >1, sum =
a[rn - 1]/
r - 1,

where n = 4, r = 2, a = 4.

Then 4[24 - 1]/2 - 1 = 4 x 15/1 = 60.

[a (1 - rn)]
8. The correct answer is option [B]. Solution: Use the equation Sn = /[1 - r], since
the common ratio is less than 1 and a = 5, r = -0.5, n = 10.
9. The correct answer is option [B]. Solution: Un = a + [n - 1]d; U8 = 2U4; U8 = a + [n - 1]d,

where n = 8; 2U4 = 2[a + (n - 1)d], where n = 4; a + [8 - 1]d

= 2[a + (4 - 1)d]; a + 7d = 2[a + 3d]; a + 7d = 2a + 6d; 7d - 6d = 2a -a; d = a………[i];

M
O
C
S.
ER
IV
G
BS
JO
Sn = [n 2a + (n -1 )d]
/2; S5 = [5 2a + (5 - 1)d]
/2 = 47; 2.5[2a + 4d]; recall that a = d,

therefore, 2d + 4d = 47/2.5; 6d = 18.8; d = 18.8/6 = 3.13.

10. The correct answer is option [C]. Solution:

The 4th term = a + 3d;

M
The 10th term = a + 9d;

O
a + 3d = 13 ------ [i];

C
a + 9d = 31 ------ [ii];

Subtract equation [i] from [ii]


S.
ER
-6d = -18; d = 3;
IV

a + 3[3] = 13;
G

a = 13 - 9 = 4
BS

Therefore, the 21st term = a + 20d = 4 + 20[3] = 64.


JO

11. The correct answer is option [C].

12. The correct answer is option [C]. Solution:

The broken lines = < or > while the unbroken lines = ≤ or

The unshaded portion is the required region.


13. The correct answer is option [C]. Solution: The length = L and breadth = B. Area =
68cm2

L x B = 68
68
B= /L

The perimeter = 2[L + B] = 42 L + B = 21.

M
O
C
S.
ER
IV
G
BS
JO
68
Substitute the value of B into the perimeter equation /L+ L = 21

68 + L2 = 21L

L2 - 21L + 68 = 0. Solve for L.

L [L - 17] + 4[L - 17] = 0

[L - 17][L + 4] = 0

L = 17 or -4.
68
The value of L = 17cm since there is no negative length. B = /L = 68/17 = 4cm

M
O
C
14. The correct answer is option [C]. Solution: Substitute the values of x into the
equation to solve for y.
S.
ER
IV

15. The correct answer is option [C].


G

n [2a + (n -1)d]
16. The correct answer is option [A]. Solution: Using L = /2, where L = 1400, n
BS

[15 [2a + (15 - 1)d] [2 1400]


= 15, a = 10; 1400 = /2 /15 = [2(10) + 14d]; 186.67 = 20 + 14d; 14d =

186.67 - 20, therefore, d = 166.67/14 = 11.90.


JO

17. The correct answer is option [D]. Solution: a = 12, r = 1/2. Using the equation S = a/ [1
- r] = 12/ [1 - 1 2] = 12/0.5 = 24.

18. The correct answer is option [B]. Solution: Common ratio, r = 12/36, 4/12 = 1/3 and a =
36, therefore, Tn = ar [n - 1] = 36[1/3] n -1 = 12n -1.
19. The correct answer is option [A]. Solution: Sixth term = a + 5d; Twelfth term = a +
11d 2[a + 5d] = a + 11d, since the twelfth term is twice the sixth term 2a + 10d = a +
11d; 2a - a = 11d - 10d; a = d = the common difference.

20. The correct answer is option [A]. Solution: a = 4, d = 2, n = 22. Sn = [n [2a + (n - 1)d]
/2

= [22 2(4) + [22 - 1](2)]


/2 11 [8 + (21)2] = 11 [8 + 42] = 11 50 = 550.

M
O
C
S.
ER
IV
G
BS
JO
CURRENT AFFAIRS

1. In _, the first Aeroplane in Nigeria landed in Kano State.

A. 1925

B. 1937

M
C. 1961

O
D. 1955

C
S.
ER
2. Heathrow Airport is a_ based airport.
IV

A. London
G

B. Paris
BS

C. Germany
JO

D. China

3. The full meaning of the acronym KLM Airline, is Koninklijke Luchtvaart Maatschappij, also
known as_ _.

A. Lufthansa German Airlines

B. Pan African Airlines

C. Kenya Airways
D. Royal Dutch Airlines

4. President Goodluck Jonathan approved the removal of the Director-General of the


Nigerian Civil Authority from office with effect from_ .

A. March 12, 2013

B. March 5, 2013

M
O
C. March 13, 2013

C
S.
ER
IV
G
BS
JO
D. March 11, 2013

5. In March 2013, Engr. Mazi Nnamdi Udoh was the Managing Director of_ .

A. Murtala Muhammed International Airport (MMIA)

B. Nigerian Airspace Management Agency (NAMA)

M
C. International Air Transport Association (IATA)

O
D. GE Capital Services Limited (GECAS)

C
S.
ER
6. The first people to visit the moon are .
IV

A. Americans
G

B. Russians
BS

C. French
JO

D. Indians

E. None of the above

7. Which of the following is NOT an example of an intergovernmental organization (IGO)?

A. The World Trade Organization.

B. Amnesty International.

C. The United Nations.


D. NATO.

8. Petroleum is the most widely used source of power because it _.

A. is relatively cheap to mine

B. is easily transported

M
O
C. yields many types of fuel

C
D. burns easily

S.
ER
IV
G
BS
JO
E. All of the above

9.. General Sani Abacha, a past military ruler of Nigeria died on .

A. May 2, 1996

B. August 15, 1997

M
C. Februaury 30, 2000

O
D. June 8, 1998

C
S.
ER
10. The Presidential system of government ceased to exist on the 31st of November 1983.
IV

A. True
G

B. False
BS
JO

11. 'Asmara' is the capital of which of the following

African countries?

A. Malawi

B. Eritrea

C. Burundi

D. Ethiopia
12. Mount Kilimanjaro is in _.

A. Tanzania

B. Kenya

C. Tunisia

D. Togo

E. Ghana

M
O
C
S.
ER
IV
G
BS
JO
13. The currency used in Israel is .

A. Lira

B. Shekel

C. Dinar

D. Rupee

E. None of the above

M
O
C
S.
14. In which of the following countries can 'Zanzibar' be found?
ER
A. Ethiopia

B. Guinea Bissau
IV

C. Tanzania
G
BS

D. Liberia
JO

15. are important groups to which individuals belong or hope to belong, that are
used as a standard for evaluating themselves, and with whose norms individuals are likely to
conform.

A. Reference groups

B. Pressure groups

C. Peer groups

D. Status groups
E. Tie groups

16. In _, Major Chuwkuemeka Nzeogu led a military coup against the Nigerian First
Republic.

A. January 15, 1966

B. March 7, 1965

M
C. August 23, 1964

O
C
S.
ER
IV
G
BS
JO
D. November 9, 1967

17. Nigeria experienced military rule for _years.

A. 27

B. 14

M
C. 15

O
D. 28

C
E. 29
S.
ER
IV

18. Sullivan Chime, in September 2012, was the Governor Of_ _.


G

A. Ebonyi State
BS

B. Enugu State
JO

C. Imo State

D. Anambra State

19. The purpoted winner of the 1993 Presidential elections, M.K.O. Abiola was arrested and
detained on .

A. June 23, 1993

B. June 12, 1994


C. June 23, 1994

D. June 12, 1993

20.. The first female political activist in Nigeria is .

A. Mrs Margraet Ekpo

M
B. Dr. Doyin Abiola

O
C
C. Mrs Bisi Ugowe

S.
ER
IV
G
BS
JO
D. Mrs Stella Obasanjo.

21. When did Ahmadu Giade complete his tenure in office?

A. November 24, 2015.

B. December 16,2018

C. March 1,1999

D. July 2, 2003

M
O
22. Col. Muhammad Mustapha Abdallah rtd was appointed by

C
A. President Muhammadu Buhari GCFR.
S.
ER
B. Shehu Hagari

C. Yemi Osinbanjo
IV

D. Dino Melaye
G
BS

23 Who’s the current Chairman/Chief executive of the NDLEA


JO

A. Ahmadu Giade

B. Mustapha Umar

C. Umar Sadiq

D. Col. Muhammad Mustapha Abdallah

24 What is the full meaning of NDLEA

A. NATIONAL DRUG LAW EQUIPMENT AGENCY

B. National Drug Law Enforcement Agency (NDLEA)

C. NIGERIAN DRUG ENFORCEMENT AGENCY


D. NATIONAL DRUG ECONOMIC AGENTS

25 What does the promulgation of Decree Number 48 of 1989, now Act of Parliament
state

A. Confiscation of illict drugs

B. Exterminating illicit drug trafficking and consumption in the Nigerian society

C. Allows the importation of drugs

M
D. Grants users amnesty

O
C
S.
ER
IV
G
BS
JO
26 It is a well-known fact that any involvement in drugs, especially their importation,
exportation, sale, transfer, purchase, cultivation, manufacture, extraction and possession
can lead to?

A. Death penalty

B. Confiscation

C. Freedom

D. Human Right

M
O
27 Who enacted the Dangerous Drugs Ordinance law of 1935

C
A. Major General Aguiyi Ironsi
S.
ER
B. Major General Murtala Mohammed

C. British Colonial administration.


IV

D. General Olusegun Obasanjo


G
BS

28 Who enacted the Indian Hemp Decree No.19 of 1966?


JO

A. Goodluck Ebele Jonathan

B. General Sani Abacha

C. General Ibrahim Babangida

D. Major General Aguiyi Ironsi

29 What was the consequence for The Indian Hemp Decree No.19 of 1966 for trafficking
A. 6 Years imprisonment

B. Bail

C. life imprisonment

D. death by firing squad

30 The Indian Hemp(Amendment) Decree No.34 of 1975 enacted by

M
A. Major General Aguiyi Ironsi

O
B. Major General Murtala Mohammed

C
C. General Ibrahim Babangida

D. Goodluck Ebele Jonathan S.


ER
IV
G
BS

ANSWERS
JO

1. A 2. A 3. D 4. A 5. B

6. A 7. A 8. C 9. D 10. B

11. B 12. A 13. B 14. C 15. A

16. A 17. E 18. A 19. B 20. A


21. A 22. A 23. D 24. B 25. B
26. B 27. C 28. D 29. C 30. B
M
O
C
S.
ER
IV
G
BS
JO

For More Exam Past Questions & Answers, Latest Job Vacancies & to Help You Source and Apply for
Jobs and Also Connect You with Employers Visit: www.jobsgivers.com Inq: 08083053951
MORE PAST QUESTIONS ON NON-MEDICAL,
SCIENCE & OTHER HEALTH RELATED POSITIONS

1. Which of the following is not true about semi-conductor? (a) moving holes are equivalent to moving
positive charges (b) there is two kinds of charge carried a free electron and a hole (c) the escape of a valence
electron from an atom produces electron hole pair charge carries (d)increases in temperature increases its
electrical resistance.

2. The minimum energy necessary to remove an electron from a given atom to infinity is called
(a) Excitation energy (b) ground state energy (c) ionization energy (d) binding energy

3. Find the proglie wavelength of a 0.0kg pellet having a velocity 10m/s (h 6.63 x 1034S) (a) 6.63-34m (b) 6.63

M
x 10-32m (c) 6.63 x 10-33m (d) 6.63 x 10-35m
4. A step transformer is designed to operate from a 25V supply. If the transformer is 80% efficient,

O
determine the current in the primary coil when the output terminals are connected to 240V 1 cow lamp (a)

C
50A (b)4.0A (c)2.5A (d)2A S.
5. An object of mass 0.2kg and density 600kgm-3 is suspended with a string so that 1/10 of it is
immerged in paraffin of density 900kgm-3, find the tension in the string (a) 1.0x105mn4 (b) 2.0 x 105ms4 (c) 3.0
ER

x 105ms4 (d)5 x105ms4

6. A rocket burnt fuel at the rate of 20kgs4 and eject it with a velocity of 5.0 x 1 03ms-1 calculate the
IV

thrust exerted by the as on the rocket. (a)1.0 x 105mns4 (b) 2.0 x 105ms4 (c) 3.0 x 105ms4 (d) 105ms4
G

7. Which of the following pairs co .ndation quantities only? (a)Velocity and gravitational potential
acceleration and field strength (c) momentum and work done ment and mass.
BS

8. One of the limitation of Thomson‘s model of the atom is that it does not explain (a) small angle scattering
(b) stability of scattering (c) ionization process (d)the variation of the effectively atomic radius
JO

1. A wire carrying a current J 2.5m in length is placed in a field of flux density 0.12T. What [ne force on the wire
if it is placed at 60° to the field? (a) 30.3N (b)20.5N (0)1 5.3N (d) 10.5N
2. In the transformer. the magnetization of the core is repeatedly reversed by the alternating magnetic field
resulting in energy dissipation as heat This loss is called (a) Eddy currents loss (b) Hysteresis loss (C) Flux
linkage loss (d) joule heating loss

9. An atom is said to be exited if an election of the atom is (a) in the ground state (b) at infinity (C)
promoted to an energy level higher than its original level (d) having energy value of 0.0cV

10. What happen to a proton number Z and the nucleon number of a nuclide which emits a y-radiation? (a) Z
increases by 1 while A does not change (b) increases by 1 and A increases by I (c)Z and A neither increases nor
decrease (d) Z increase by 1 while A decreases by 1

11. A note that is an OCTAVE higher than a given note of frequency 25hz world has a frequency of (a)
2048Hz (b) 1024z (c) 12Hz (d) 512Hz
12. A sound wave of the frequency 130Hz and wavelength 2.0m was sproceeded at a distance. d. from a
target and echo was heard at the source 0.5second later. Calculate the values of d (a) 650m (b) 1300m (c)260.0m

M
(d) 520.0m
13. In an experiment, lead shot contained in a vertical cardboard cylinder falls through 100cm when the

O
cylinder is invested. Calculate the rise in temperature caused by 100 such inversion. (Specify heat capacity of
lead 1 3jkg-1k-1g = 10ms-2 (a) 3.3K (b)4.4K (c)5.5K (d)7.7K

C
14.
S.
The QUALITY and PITCH of musical note depend respectively on (a)frequency and harmonies (b)
overtones and intensity (c) intensity and frequency (d) overtones and frequency
ER

15. A body is project with a velocity with a velocity Vms-1, inclined at angle B to the vertical. Which of the
following give the CORRECT expression for the horizontal component of velocity Vx after time t? (a)V cos B
IV

(b) Vsin B (c)Vg cos B(d) Vtsin B


G

16. A converging lens has radius of curvature 16.0cm, calculate the power of the lens (a) 0.0625 (b)0.
125 (c) 0.250 (d)0.500
BS

17. If the velocity of light in air is 3.0 x 108ms, find the velocity of light in s medium whose refractive index is
1.5 (a)5.0x108ms4 (b)5.0x107ms4 (c)2.5x108ms4 (d)2x107ms4
JO

18. When an object is at infinity to the pole of a concave mirror the image formed is at (a) principle focus,
real and diminished (b) center of curvature, real and inverted (c) center of curvature, virtual and erect
(d)principle focus, real and erect

19. A total eclipse of the sun is seen when the observer is in the (a) Umbra region of the sun‘s shadow (b)
penumbra region (C) Umbra region of the moon‘s show (d) penumbra region of the moons shadow.

20. A long drawn sound arising from overlapping reflections together with the original sound which give rise
to the reflections is called (a) Beat(b) Echo (c) diffraction (d) reverberation

21. Which of the following radiation is used for examining the freshness or staleness of eggs in the poultry (a)
Ultraviolet rays (b) X-rays (c) Y-rays (d) radio waves

22. Which of the following statements is not correct about magnetic radiation? (a) they are quantities en (b) they
are transverse waves (c)they are polarizable (d) they may be deflect in an electronic or a magnetic field
ONDO SCHOOL OF HEALTH PAST QUESTIONS AND ANSWERS 2010-2018
23. A simple cell with amalgamated zinc plate minimizes (a) sparking (b) local (c) Back e.m.f (d)
polarization

26. Which of the following mixture? (a) sea water (b) petroleum (c) urine
(d)

27. Carbon forms two oxides in both oxides is 1.2 if one mole absorbed separately combines with oxygen,
deduces of the oxides (a) CO;CO2 (b)CO2CO3 (c) CO2C2O (d) CO2CO3

28. If 50cm3 of CO2 gas and 50cm3 of measured, at S.T.P calculate the amount gas in moles (molar gas
volume 22.4 dat) (a) 0023mo1 (b) 0.0222mo1 (c)O.0002moI (d) 0.0022mo1

29. How much volume of the ethane would be required to produce 1.12dm3 of carbor1 (IV)on combustion in
sufficient oxygen? 2c2 H6(g)+ 6)2 (G) 4C02(g) + 6H2O (a) 56dm3 (b) 2.24dm of ethane (c)056dm3 of ethane
(d)56dm3 of ethane
By how much would the volume have increased when 10dm3 of ozone were converted to oxygen 203(g)

M
30.
3002(g) (a) 15dm3 (b)19dm3 (c)0.5dm3 (d)5dm3

O
31. The pressure excreted a gas is as a result of (a) the continuous rand on motion of its particles (b)

C
Bombardment of the walls of the container by its molecules (c) the collision between the gas molecules (d)the
elastic nature of the gas molecules
S.
32. Which of the following gas will diffuse at the slowest rate? (a)Ammonium (b) Sulphate (IV) oxide (c)
ER

Carbon (II) oxide (d) Nitrogen

33. Element P has an electron configuration of 2.8.6. Element R has an electronic configuration of 2.8.8.1
IV

What is likely to form if P and R COMBINE? (a) A covalent compound UPR (b) An ionic compound PR2
(c)An ionic compound PR (d)An ionic compound P2R
G

34. Which is the atomic structure of phosphorus with mass number 31?
BS

Protons Neutrons Electrons


A. 15 16 15
B 15 16 16
JO

C. 16 15 15
D. 16 15 16

35. Which metal has the least tendency to form positive ions? (a) Iron (b) Aluminum (c) Sodium (d) Calcium

36. Natural water includes the following except (a) rain water (b) spring (c)pure water (d) lake water

37. Which of the following methods cannot be used to remove permanent hardness in water? (a) addition of
washing soda (b) addition of caustic soda (c) permutit method (d) adding alum to water

38. If the solubility of sodium tetraoxosulphate (IV) at 30°C is 18g per 100g, how much is this in
gramme per kilograms? (a)18kg per 100g (b)180kg per 100g per 100g (c) l80per 100g (d)180g per 100g
ONDO SCHOOL OF HEALTH PAST QUESTIONS AND ANSWERS 2010-2018
39. The following are examples of colloid except (a) milk (b) starch in water (c) aerosol spray (d) ammonium
chloride solution

40. The pH of the solution M,N,O, and P are 46,8 and 10, respectively, therefore (a) none of the solutions
is acidic (b) the pH of 0 is made neutral by adding water (c) P is the most acidic solution (d) M is the most
acidic solution

41. Sodium chloride may be obtained from brine by (a) decantation (b)distillation (c) evaporation (d)
sublimation

42. Oil spillage in ponds and creeks can be cleaned by (a) Burning of the oil layer (b) spraying with detergent
(c) spraying with common salt (d)spraying with hotoil

43. Which of the following is a chemical compound? (a) soap (b) milk (c)urine (d) gold

44. Crystallization is a separation method used (a) where purity of the product is important (b) where

M
beauty of the product is important (c)where one of the product is a solid (d) where the salt cannot be

O
destroyed by heat.

C
45. Which hydroxide dissolves in water to form an alkaline solution (a)aluminum hydroxide (b) copper
hydroxide (C) copper (II) hydroxide (d) Iron (III) hydroxide
S.
46. Which of the following cannot be used as an oxidizing agent? (a)cracking (b) hydrolysis (c) oxidation
ER

(d) reduction

47. Which of the following cannot be used as an oxidizing agent? (a) K2Cr2 (b)CO2 (c) H2H (d) HNO3
IV

48. What is the number of the pubs of shared electrons in a methane molecule? (a) I (b)2 (c)4 (d) 8
G

49. The percentage by volume of nitrogen in air is high because (a) nitrogen is relatively inactive (b) nitrogen
BS

supports life (c) nitrogen prevents corrosion of metals (d) nitrogen increases the rate of combustion

50. What quantity of copper will be deposited by the same quantity of electricity that deposited 1 8g of
JO

aluminum (A1 = 27, Cu = 64) (a) 64g (b)32g(c) 16g (d)8g

51 When two genes for the same character (alleles) are contained n the same individuals, the character
that shows is known as (a) important character (b) dominant character (c) superior character (d) controlling
character

52. In Nigeria a tropical rain forest can be found in (a) Sokoto (b) Kaduna (c)Abuja (d)Abia

53. The sum total of the biotic and abiotic factors that affect living thing is referred to as: (a) environment (b)
lithosphere (c) hydrosphere (d)atmosphere

54. Hydrometer is an ecological instrument that, measures (a) rainfall (b)humidity (c) temperature (d)
light

55. Which one of the following is air borne? (a) malaria (b) yellow fever (c)cholera (d)tuberculosis
ONDO SCHOOL OF HEALTH PAST QUESTIONS AND ANSWERS 2010-2018
56. Candida virginalis is a (a) bacterium (b)fungus (C) virus (d) protozoan

57. All of these are vertebrate except (a) lizard (b) rat (c) star fish (d) tilapia

58. A box was left in the lawn for two days. When the box was removed, the grass under had turned yellow due
to lack of (a) carbon dioxide (b) light (c) oxygen (d)water

59. Which of the following deaminates excess amino acids? (a) duodenum (b) ileum (c) liver (d) kidney

60. In which of the following flower part does meiosis occur? (a) anther (b)petal (c) receptacle (d) style

61. When an organism moves its whole body towards a stimulus, the organism is said to exhibit (a)
tropic movement (b) tropic movement (c)tactic movement (d) nastic movement

62. A stable self sustaining environment produced by an interaction between the biotic and abiotic components
is best described as (a) niche (b)a community (c)an ecosystem (d)a habitat

M
63. Which of the following is not present in the nucleus of a cell? (a)chromosome (b) nucleus (c) genes (d)

O
mitochondria

C
64. The release of useful substances from cells of an organism is called? (a)excretion (b) evacuation (c)
metabolism (d) secretion
S.
ER

65. The head of the femur articulates with the pelvic at the (a) Glenoid cavity (b) Olocrono process
(c)Acetabulum (d) Caracoids process
IV

66. The mammalian cervical vertebrae invariably numb

67. What mechanism is responsible for the rise of water to the tops of tall trees? (a)transpiration er (a) 4 (b) 7
G

(c) 12 (d) 5 pulls (b) root pressure (c)osmosis (d) capillarity


BS

68. Antibodies in mammalian blood are formed by (a) platelets (b) white blood cells (c) red blood cells (d)
liver
JO

69. Which of the following liquids supplies cells in the tissues of a mammal with oxygen and nutrients? (a)
blood (b) plasma (c) serum (d) lymph

70. The cockroach and grasshoppers have mouth parts adapted for (a)sucking (b) piercing and sucking
(c) biting and chewing (d) biting and lapping
ONDO SCHOOL OF HEALTH PAST QUESTIONS AND ANSWERS 2010-2018
71. The sponging and lapping mouth part occurs in (a) butterfly (b) the cockroach (c)the housefly (d)
mosquito

72. The following is not an example of a sex-linked character (a) baldness (b)color blindness (c) hemophilia
(d) height

73. Which of these is a vestigal structure? (a) Appendix (b) caccum (c)pancreas (d) sacculus motundus

74. Which of the following air pollution causes acid rain? (a) sulphur dioxide (b) blend oxide (c) carbon
dioxide (d) hydrogen sulphide

75. Which of the following does not have a well developed tissue? (a) moss (b) fern (c) whiz pering tree (d)
76. maize

M
ANSWERS

O
1. D
2. A

C
3. C S.
4. A
5. D
ER

6. A
7. -
8 -
IV

9. -
10. -
G

11. C
12. A
BS

13. D
14. A
JO

15. D
16. D
17. A

18.
19. -
20. -
21. - 51. A
22. D 52. B
23. A 53. A
24. A 54. B
25. - 55. D
26. - 56. B
27. - 57. C
28. D 58. B
29. C 59. C
30. B 60. A
31. B 61. A
32. B 62. A
33. B 63. D
34. A 64. D
35. B 65. C
36. C 66. B
37. D 67. -
38. - 68. B
39. D 69. A
40. D 70. C
41. C 71. A
42. B 72. D
43. A 73. A
44. A 74. A
45. B 75. A
46. -
47. -

M
48. -
49. A

O
50. A

C
S.
ER
IV
G
BS
JO
20

JO
BS
G
IV
ER
S.
C
O
M

You might also like